Sie sind auf Seite 1von 431
Solutions to LE. Irodov’s Problems in General Physics Volume I Mechanics e Heat e Electrodynamics SECOND EDITION ABHAY KUMAR SINGH Director Abhay’s |.1,T. Physics Teaching Centre Patna-6 cBs CBS PUBLISHERS & DISTRIBUTORS 4596/1A, 11 DARYAGANJ, NEW DELHI - 110 002 (INDIA) Dedicated to my Teacher Prof. (Dr.) J. Thakur (Department of Physics, Patna University, Patna-4) ISBN : 81-239-0399-5 First Edition : 1995 Reprint : 1997 Second Edition : 1998 Reprint : 2000 Reprint : 2001 Reprint : 2002 Reprint : 2003 Reprint : 2004 Reprint : 2005 Copyright © Author & Publisher All rights reserved. No part of this book may. be reproduced or transmitted in any form or by any means, electronic or mechanical, including photocopying, recording, or any information storage and retrieval system without permission, in writing, from the publisher. Published by S.K. Jain for CBS Publishers & Distributors, 4596/1A, 11 Darya Ganj, New Delhi - 110 002 (India) Printed at : : India Binding House, Delhi - 110 032 FOREWORD Science, in general, and physics, in particular, have evolved out of man’s quest to know beyond unknowns. Matter, radiation and their mutual interactions are basically studied in physics. Essentially, this is an experimental science. By observing appropriate phenomena in nature one arrives at a set of rules which goes to establish some basic fundamental concepts. Entire physics rests on them. Mere knowledge of them is however not enough. Ability to apply them to real day-to-day problems is required. Prof. Irodov’s book contains one such set of numerical exercises spread over a wide spectrum of physical disciplines. Some of the problems of the book long appeared to be notorious to pose serious challenges to students as well as to their teachers. This book by Prof. Singh on the solutions of problems of Irodov’s book, at the outset, seems to remove the sense of awe which at one time prevailed. Traditionally a difficult exercise to solve continues to draw the attention of concerned persons over a sufficiently long time. Once a logical solution for it becomes available, the difficulties associated with its solutions are forgotten very soon. This statement is not only valid for the solutions of simple physical problems but also to various physical phenomena. Nevertheless, Prof. Singh’s attempt to write a book of this magnitude deserves an all out praise. His ways of solving problems are elegant, straight forward, simple and direct. By writing this book he has definitely contributed to the cause of physics education. A word of advice to its users is however necessary. The solution to a particular problem as given in this book is never to be consulted unless an all out effort in solving it independently has been already made. Only by such judicious uses of this book one would be able to reap better benefits out of it. As a teacher who has taught physics and who has been in touch with physics curricula at 11.T., Delhi for over thirty years, I earnestly feel that this book will certainly be of benefit to younger students in their formative years. Dr. Dilip Kumar Roy Professor of Physics Indian Institute of Technology, Delhi New Delhi-110016. FOREWORD A. proper understanding of the physical laws and principles that govern nature require solutions of related problems which exemplify the principle in question and leads to a better grasp of the principles involved. It is only through experiments or through solutions of multifarious problem-oriented questions can a student master the intricacies and fall outs of a physical law. According to Ira M. Freeman, professor of physics of the state university of new Jersy at Rutgers and author of “‘physic--principles and Insights’” -- “In certain situations mathematical formulation actually promotes intuitive understand- ing....... Sometimes a mathematical formulation is not feasible, so that ordinary language must take the place of mathematics in both roles. However, Mathematics is far more rigorous and its concepts more precise than those of language. Any science that is able to make extensive use of mathematical symbolism and procedures is justly called an exact science’. I.E, Irodov’s problems in General Physics fulfills such a need. This book originally published in Russia contains about 1900 problems on mechanics, thermody- namics, molecular physics, electrodynamics, waves and oscillations, optics, atomic and nuclear physics. The book has survived the test of class room for many years as is evident from its number of reprint editions, which have appeared since the first English edition of 1981, including an Indian Edition at affordable price for Indian students. Abhay Kumar Singh’s present book containing solutions to Dr. I.E. Irodov’s Problems in General Physics is a welcome attempt to develop a student’s problem solving skills. The book should be very useful for the students studying a general course in physics and also in developing their skills to answer questions normally encountered in national level entrance examinations conducted each year by various bodies for admissions to profes- sional colleges in science and technology. BP. PAL Professor of Physics LLT,, Delhi PREFACE TO THE SECOND EDITION Nothing succeeds like success, they say, Now, consequent upon the warm welcome on the part of students and the teaching fraternity this revised and enlarged edition of this volume is before you. In order to make it more up-to-date and viable, a large number of problems have been streamlined with special focus on the complicated and ticklish ones, to cater to the needs of the aspiring students. I extend my deep sense of gratitude to all those who have directly or indirectly engineered the cause of its existing status in the book world. Patna June 1997 Abhay Kumar Singh PREFACE TO THE FIRST EDITION When you invisage to write a book of solutions to problems, one pertinent question crops up in the mind that—why solution! Is this to prove one’s erudition? My only defence against this is that the solution is a challenge to save the scientific man hours by channelizing thoughts in a right direction. The book entitled “Problems in General Physics” authored by LE. Irodov (a noted Russian physicist and mathematician) contains 1877 intriguing problems divided into six chapters. After the acceptance of my first book “Problems in Physics”, published by Wiley Eastern Limited, I have got the courage to acknowledge the fact that good and honest ultimately win in the market place. This stimulation provided me insight to come up with my second attempt—‘Solutions to I.E. Irodov’s Problems in General Physics.” This first volume encompasses solutions of first three chapters containing 1052 problems. Although a large number of problems can be solved by different methods, I have adopted standard methods and in many of the problems with helping hints for other methods. In the solutions of chapter three, the emf of a cell is represented by & (xi) in contrast to the notation used in figures and in the problem book, due to some printing difficulty. I am thankful to my students Mr. Omprakash, Miss Neera and Miss Punam for their valuable co-operation even in my hard days while authoring the present book. I am also thankful to my younger sister Prof. Ranju Singh, my younger brother Mr. Ratan Kumar Singh, my junior friend Miss Anupama Bharti, other well wishers and friends for their emotional Support. At last and above all I am grateful to my Ma and Pappaji for their blessings and encouragement. ABHAY KUMAR SINGH CONTENTS Foreword Preface to the second edition Preface to the first edition PART ONE PHYSICAL FUNDAMENTALS OF MECHANICS Kinematics The Fundamental Equation of Dynamics Laws of Conservation of Energy, Momemtum, and Angular Momentum Universal Gravitation Dynamics of a Solid Body Elastic Deformations of a Solid Body Hydrodynamics Relativistic Mechanics PART TWO THERMODYNAMICS AND MOLECULAR PHYSICS Equation of the Gas State. Processes The first Law of Thermodynamics. Heat Capacity Kinetic theory of Gases. Boltzmann’s Law and Maxwell’s Distribution The Second Law of Thermodynamics. Entropy Liquids. Capillary Effects Phase Transformations Transport Phenomena PART THREE ELECTRODYNAMICS Constant Electric Field in Vacuum Conductors and Dielectrics in an Electric Field Electric Capacitance. Energy of an Electric Field Electric Current Constant Magnetic Field. Magnetics Electromagnetic Induction. Maxwell’s Equations Motion of Charged Particles in Electric and Magnetic Fields vi 1-34 35-65 66-101 102-117 118-143 144-155 156-167 168-183 184-195 196-212 213-226 227-241 242-247 248-256 257-266 267-288 289-305 306-324 325-353 354-379 380-407 408-424 PART ONE PHYSICAL FUNDAMENTALS OF MECHANICS 11 KINEMATICS 11 Let v, be the stream velocity and v’ the velocity of motorboat with respect to water. The motorboat reached point B while going downstream with velocity (v, + v’) and thea retuned with velocity (v’ - v,) and passed the raft at point C. Let ¢ be the time for the raft (which flows with stream with velocity v,) to move from point A to C, during which the motorboat moves from A to B and then from B to C. Therefore i a dak wenn --(Vot VN ----+ > ee A-——— 7 B 2% On solving we get v, = + ——1—¢ = 1.2 Let s be the total distance traversed by the point and 1, the time taken to cover half the distance. Further let 2¢ be the time to cover the rest half of the distance. s 5 Therefore Zr or @ and Ze, +v)t or =e (2) et Wy +Vy Hence the sought average velocity = % (vy + ¥) s s SP" Fed” 2 PIMs) ++ 2v, 13 As the car starts from rest and finally comes to a stop, and the rate of acceleration and deceleration are equal, the distances as well as the times taken are same in these phases of motion. Let At be the time for which the car moves uniformly. Then the acceleration / deceleration 7 a each. So, time is » 14 y evo t= 24d w EAN |, y EA) a | 2 Gy Are P@-4S2t w Hence Arm vV 1-482 0 55, wr (a) Sought average velocity s_ 200cm a2 ae 20s =10cm/s (b) For the maximum velocity, $ = ; should be maximum. From the figure # is maximum for all points on the line ‘ac, thus the sought maximum velocity becomes average velocity rc] for the line ac and is equal to : 0 10 20%S be | 100 cm 95 cm/s ab 4s (©) Time fy should be such that costing to it the slope 4S ould pass through the point O (origin), to satisfy the relationship Se z From figure the tangent at point d passes through the origin and thus corresponding time t= f) = 16s. 1.5 Let the particles collide at the point A (Fig.), whose position vector is 73 (say). If t be the 16 time taken by each particle to reach at point A, from triangle law of vector addition : Fre tyta t yt Be eth Pot A 8, ~~) @ d vie l-Fl therefore, f= ==, (2) ee => Iy2-vi| % Vat From Eqs. (1) and (2) > oe etal RR 7, $ ee eaeler | 0 z tet Vent 2 oc ee , which is the sought relationship. =m” T= : We have . vet @) From the vector diagram [of Eq, (1)] and using properties of triangle 17 va Vite? 42vgvcosp = 39.7km/br (2) : : a v eu +, p- vei f od Sop me sin ® oe ¥ v or O= sin? fA ~ Using (2) and putting the values of v and d % @=19.1° Let one of the swimmer (say 1) cross the river along AB, which is obviously the shortest path. Time taken to cross the river by the swimmer 1. da he ve For the other swimmer (say 2), which follows the quickest path, the time taken to cross the river. , (where AB = d is the width of the river) 0) In the time £,, drifting of the swimmer 2, becomes y X= Vol = yh (using Eq. 2) @) If f, be the time for swimmer 2 to walk the distance x to come from C toB (Fig.), then x_ vod . = 57 yg (using Ea. 3) 4 u According to the problem f; = f+ f3 & or, On solving we get 4 18 19 Let I be the distance covered by the boat A along the river as well as by the boat B acrc the river. Let vy be the stream velocity and v’ the velocity of cach boat with respect water. Therefore time taken by the boat A in its journey I 4" Jan Von (eee BV Via Vo and for the boat B 4X Vv We Hence, 4. 4. — 1 | wheren = 4 te Vv7-v Vae=-1 ae On substitution ty/ty= 18 Let vp be the stream velocity and v’ the velocity of boat with respect to water. A ¥ or 1 = 2>0, some drifting of boat is inevitable. Let v” make an angle 0 with flow direction. (Fig.), then the time taken to cross the rive d , . t= Tang (where d is the width of the river) In this time intezval, the drifting of the boat x= (v' cos 6+ v9) t = (0/008 04 vq) 4 = (cot +n cosec 8) d | —_— For qj, (minimum drifting) ae d d No _ Fo (cot 0 + n cosec 6) = 0, which yields y ; i , — ooso0 te} 6 n 2 x Hence, = 120° 0 The solution of this problem becomes simple in the frame attached with one of the bodies. Let the body thrown straight up be 1 and the other body be 2, then for the body 1 in the frame of 2 from the kinematic equation for constant acceleration : ls 2 mh" Toa) * Yount + > Mi2t So, Tham Vous (because Wig= 0 and Fo2)= 0) or, Fial= Ioaalt @ But |¥q1= ll % So, from properties of triangle Yoa2) = V vB + v2 -2 v9 v9 cos (/2 - 8) Hence, the sought distance |7yl= vo V2 -sin ®) t= 22m. 1 112 5 Let the velocities of the paricles (say vy and ¥]_) becomes mutually perpendicular after time ¢. Then their velocitis become me ee wet oe Vy = Vj +Bt; vy = v2 tee @ As vy Lip 80, vv) = 0 or, (+84): (+84) = 0 or -yy y+ t= 0 Hence, t= “_ 1 Now form the Eq. Fi = Taz + Youn! + zh ot l7yl= [Vaal (because here Wi, = 0 and 792) 9) Hence the sought distance mi = +2 => ale AEBV i Galtian l= v4 Yd) From the symmetry of the problem all the three points are always located at the vertices of equilateral triangles of varying side length and finally meet at the centriod of the initial equilateral triangle whose side length is a, in the sought time interval (say ¢). > = 120° 3° Let us consider an arbitrary equilateral triangle of edge length / (say). ‘Then the rate by which 1 approaches 2, 2 approches 3, and 3 approches 1, becomes : On integrating : 6 113 1.14 Let us locate the points A and B at an arbitrary instant of time (Fig.). If A and B are separated by the distance s at this moment, then the points converge or point A approaches B with velocity =4. v—ucos a where angle o. varies with time. On intergating, o ae ~fds= fv-ucos a) at, - o (where T is the sought time.) r or I= f(v-ucos.a) dt ) 0 As both A and B cover the same distance in x-direction during the sought time interval, so the other condition which is required, can be obtained by the equation xm fv,at So, 'v cos «1 de 2) 0 Solving (1) and (2), we get T= wo -u One can see that if u= v, or u D, and the time taken to travel the distance CB A in the field a vise ® “A Yr So, the total time elapsed to move the car from point A to B ad 1 AD-x Vex | aa ee gee eee nV v For ¢ to be minimum \ B Ce Oe de vin VP er or Wee Pa or x= 9 1.18 To plot x(#),s(¢) and w, (¢) let us partion the given plot v, (f) into five segments (for detailed analysis) as shown in the figure. For the part oa: w,= 1 and v,= t= v t 2 Thus, Ax, ()= fu,de= far= a 5,0 0 5 Putting t= 1, we get, Ax, = s= 5 unit For the part ab : w,= O and v,= v= constant = 1 ‘ Thus An (= frpdt= fat= ¢-1)= 40 1 Putting t= 3, Ax = s)= 2unit For the part b4: w,= 1 and vy,= 1-(t-3)=4-H)=¥ ‘ 2 ita 15: Thus Oe (0) Putting t= 4, Ax,=x3= unit For the part 4d : v= -Land v= -(1-4)= 4-1 So, ve |yf=t-4 for t>4 ‘ 2 ie Thus CTO ULE 1p Putting t= 6, Ax,=-1 : 2 sauat - Similarly Oa eater Putting t= 6, s,= Qunit For the partd7: w,= 2 and v,= -2+2(t-6)= 2(¢-7) vel ‘ Now, Ax(t)= [2 @-Tdt= 17-144 48 : Putting t= 4, Arye -1 6 Similarly 55()= [2(7-1)de= 144—17~ 48 : Putting t= 7, s5=1 On the basis of these obtained expressions w, (2), x(t) and s(t) plots can be easily plotted as shown in the figure of answershect. 10 119 1.20 (a) Mean velocity Total_distance covered Time elapsed -i- aR. S0cm/s (1) Vo (b) Modulus of mean velocity vector i f : <»,_ |Ar]_2R [= - . 3 “3 « sa o So, from (1) and (3) Yotv aR 2 7% c Now the modulus of the mean vector of total acceleration oe [Avy Il Mote : || Wi : 7 (see Fig.) S Using (4) in (5), we get : : =. R ||- == (a) we have r= at(l-at) So, ve #. aU -2a8 — and we & =-2aa" (b) From the equation 7= 0, at t= O and also at t= At= 2 So, the sought time Ar = i As v= a(1-2as) a(1-2a0) forts x So, v= |vy= ; a(2at-1)] fore> x5 By Hence, the sought distance 1a Va s= fvde= f a(1-2andr+ f aQar-1a 0 va Simplifying, we get, s= aL (a) As the particle leaves the origin at f= 0 So, Arex= fo.de qa) ft As ve (1-7), where vo is directed towards the +ve x-axis So, v= V9 (: - ‘} 2) From (1) and (2), vo fn(t-dae roe(t-34} ) 0 Hence x coordinate of the particle at r= 6s. x= 1x6 {1-305 a)" 24em = 024m Similarly at t= 10s 10 x= 10%10(1 3205} 0 and at t= 20s 20 x= 10% 20(1-32%5) -200cm= -2m (b) At the moments the particle is at a distance of 10 cm from the origin, x = + 10cm. Putting x= +10 in Eq. (3) =e) a a 10= 10:(1 75} on, 1?~10t+10= 0, So, te r= 102 Y100= 40 VO 52 Vs Now putting x= -10in Eqn (3) t -10= 10(1-i5} On solving, t-52VB s As t cannot be negative, so, t=(5+V35)s 12 1.22 Hence the particle is at a distance of 10 cm from the origin at three moments of time : t=S2Vi5 s,5+v35 s (c) We have ve voli -= We hi ve [1 -F £ vo(t-3) forts t So, ve |vy= v,(£-1)} fort>t ls So se vo(t-z)a for ts t = vgt(1- 4) «ote ' t t and on foft-s)arefult-s)a for t> 0 t = vt [1+ (1- 44/2 for t>e (A) 4 4 t t on fn(i-ga- frof-ga- 24cm. o 0 And for t= 8s 7 8 t t oo f0(t-Jae frogs} 0 5 On integrating and simplifying, we get s= 34cm. On the basis of Eqs. (3) and (4), x (¢) and s(t) plots can be drawn as shown in the answer sheet. As particle is in unidirectional motion it is directed along the x-axis all the time. As at t=0,x=0 dv So, Av=x= \d = » x= sand 7 Therefore, v= avx = avs dv a_ds a or, we = = oe xe a dv a As, a 2 | a a On integrating, fo- Fa o, ve St (2) 13 (b) Let s be the time to cover first s m of the path. From the Eq. s= fvdt ; oe a? 1? . fe ae SE (using 2) 0 or t= V5 @) The mean velocity of particle 2Vs/a f e tdt froa 7 ava la Ne? 1.23 According to the problem - vd, avy (as v decreases with time) 9. ‘ or, -f Wav= afas Yo 0 : : 2 32 On integrating we get s= —v} 3a Again according to the problem 1.24 (a) As P= atizbey” So, x= at, y=-bt? _ bx? and therefore y= @ 14 1.25 which is Eq. of a parabola, whose graph is shown in the Fig. (b) As Fe atizbr jy” a ve a ait absy So, v= Va7(-2bt) = Va7+4br Diff. Eq. (1) watt. time, we get a dv" o> We Pm - 2] So, [Wl =w=2b Ww (i= 2617) (-26]T ©) cos a= eee aa (Va? +4671?) 2 2bt or, cos @ = Va7+4b7 17 a 50, tana= 557 -if_@ or a= ar*( ) (d) The mean velocity vector fre flarapejsae o> 0 = Ja -—— Hence, [ |= Va7+ (=b1)? = Va74b7 = aizbtj” (a) We have x= atand y= at(l-at) Hence, y (x) becomes, a rrrC—S yn (1 =) x- 2x? (parabola) (b) Ditferentiating Eq. (1) we get v,= a and v,= a(1-2at) q@ @) @ 1.26 127 15 So, v= Wr 4yy =aV14(1-208)7 Diff. Eq. (2) with respect to time w,= O and w= -2a0 So, wa Vat w= 2aa (©) From Eqs. (2) and (3) We have Ve ajta(1-2at)j” and W= 2aaj” yw -a(1-2a%)2 So, cote fa BH (1-244)2a0 4 v2 vw aV1+(1-20%) 2aa On simplifying. 1-2a= 21 i As, n0, hee Differentiating motion law : x= asinwt, y= a(1-cosm@r), with respect to time, V, = @0) COS, v, = a@sinot So, V= awcosot{+ awsinwt i” qa) and v= a@= Const Q) Differentiating Eq. (1) with respect to time > we e. -aw*sinotitaw?coswtj- @) (a) The distance 5 traversed by the point during the time v is given by s=fvde= fawde= aor (using?) 0 0 (b) Taking inner product of v” and = ri ot a : 2 a We get, Vv? W= (a cos wti+asin tj) (aw sin wt (-i) +a w* cos wt -j) So, v" We - a? wo sin wt cos wt + a” w’ sin wt cos wt = 0 Thus, 7°L W, i.e., the angle between velocity vector and acceleration vector equals i Accordiing to the problem we w(-Jd dy, dy, =. tee So, w= Git 0 and w= = -w Q) Differentiating Eq. of trajectory, y= ax—- bx”, with respect to time dy adx_4,, de qe Gp 2bx Fe 2) 16 1.28 dy So, ae at jx=0 Again differentiating with respect to time Pye 1220/2) -26 ax ae de dt ai? dx ' or, yO). 20(G y ~2bx (0) (using 1) dx w - or, EV (using 1) ¢ ae dy) fe Using (3) in (2) ay |... aVvz « Hence, the velocity of the particle at the origin veV ()., (Z we a? (using Eqns (3) and (4)) Hence, ve Vi 1+0) As the body is under gravity of constant accelration °, it’s velocity vector and displacemen vectors are: a ve =% +B Q and Are r= weed gt? (= Oats = 0) (2 So, over the first ¢ seconds > A 7» Be Are Hae @) Hence from Eq. (3), <> over the first t seconds ~ <= ge (4) For evaluating t, take VVE (9) + BH + BD) = VR+2 (Mots et? or, = vet (ipRt+ gt? But we have v= vp at ¢= O and Also at f= + (Fig.) (also from energy conservation) ho 17 Hence using this propety in Eq. (5) vem va+2(vpge+ gr? 20-8 As 120, so, t= -—5 g Putting this value of t in Eq. (4), the average velocity over the time of flight gsinat 2v) 4 2v9 50, I= vpsin a. |—] += sina |— : a) 28 G 4 v2 sin o = —2— (using 2) Hence the sought distance, I= SGg)sne. Shsina (Using Eq. 1) Total time of motion 2 vp sino : ee ee gg ee 2¥% 2x 240 @ and horizontal range Rx vcosat or cosa~ . $100, 85 : vot 240% 41 Q) From Eqs. (1) and (2) (arr? | 5" | (480) (4 2 2 On simplifying v4 — 2400 x? + 1083750 = 0 20 1.33 134 Solving for t” we get : 2 2400 + V 1425000 2400+ 1194 2 Thus = 4239s= 0-71 min and t= 2455s = 0-41 min depending on the angle a. Let the shells collide at the point P (x, y). If the first shell takes f s to collide with second and Af be the time interval between the firings, then X= vy cos 0, t= vy cosO,(t-At) (1) and y= vpsin 04-3 gt? y¥ : . = vosin 8, (¢-A)-Fe- A @) [aR At cos 0, (ay) From Eq.) t= Sa aaeo @) P From Eqs. (2) and (3) RK 4 0 2 vp sin (0, - 0,) x Ate OT as Ate 0 & (cos 0, + 00s ,) According to the problem (a) 2. vp or dy= vpdt y Integrating Jar vof dt or y= vot (ay 0 0 And also we have a. ay or de= aydt=avytdt (using 1) : A 2 So, fae avy feat, or, x= davyr?= 122 using 1) 2 2% oO 0 () According to the problem v= vy and v= ay (2) So, va Vibe Vibe? 2 2 Therefore w= % a SY ey a Vijray dt” Vy + (ayy Diff. Eq. (2) with respect to time. dy dy dv, = w,= 0 and [A= w= a= av So, 135 1.36 “Vwow= V ev .—ri Integrating this equation from vys vs v and Os ss s - s 1 Ves so, fsb few weet i 0 ’ Hence ve we (2) (0) The normal acceleration of the point 2 -2s/R “= Re ace (using 2) And as accordance with the problem |wjl= |w,| and wi, Lw, te, 2 2 Yo -2/R v so, w= V2w,= v2 pe WR 23 1.39 From the equation v= avs As w, is a positive constant, the speed of the particle increases with time, and the tangential acceleration vector and velocity vector coincides in direction. Hence the angle between v°and wis equal to between w,i#, an W, and a. can be found I¥,1_ a?s/R_ 2s means of the formula : tana = = == u Tw" aa 1.40 From the equation I= asinwt a, v= awcoswr at So, w= 4. ~actsinor, and (yy @ w’ cos’ wt Q (a) At the point ]= 0,sin@t= 0 and cosmf= + 1 so, w= 0, 2 etc. 22 @ ow Hence w= w= Similarly at/= + a, sinwr= + 1 and cos@t= 0, so, w,= 0 Hence we |w,|= ao" 1.41 As w,= a and at ¢= 0, the point is at rest So, v(t) and s(t) are, v= at and s= fat? q@ Let R be the curvature radius, then 2 me oe oo RO RR (using 1) But according to the problem w, = bt* 2.2 2 a a asi So, bt*= R Zhe (using 1) (2) Therefore w= Vwe+w? = Va?+(2a5/R) = V a + (4 bs? / a?) (using 2) Hence we aVis (4087/0) 24 1.42 (a) Let us differentiate twice the path equation y (x) with respect to time. at’ gy dt Since the particle moves uniformly, its acceleration at all points of the path is normal and at the point x = 0 it coincides with the direction of derivative d?y/dt?, Keeping in mind 2 2 #. r0H; Ye 24 2|() vel that at the point x= 0, |@ =y oy tate Wp We get we |B z= 0 2 So, wo 2avte td, or Re Note that we can also calculate it from the formula of problem (1.35 b) (b) Differentiating the equation of the trajectory with respect to time we see that Bx a, ay no ey) which implies that the vector (b’xi"+ a”yjJ is normal to the velocity vector v= & i+ 2 which, of course, is along the tangent. Thus the former vactor is along the normal and the normal component of acceleration is clearly dx ody Bx + ay at ae Wa ae (B+ a) on using w,= wen/|n'].Atx= 0, y= = band soatx=0 Differentiating (1) 2 2 Hafolghaah-ath Also from (1) Peonx-0 So (@)- v (since tangential velocity is constant = v ) Thus (2)- * ae wiv = [ml En This gives R = a/b. 143 1.44 25 Let us fix the co-ordinate system at the point O as shown in the figure, such that the radius vector 7 of point A makes an angle @ with x axis at the moment shown. Note that the radius vector of the particle A rotates clockwise and we here take line ox as reference line, so in this case obviously the ' de® e angular velocity o-= on taking anticlockwise sense of angular displacement as positive. Also from the geometry of the triangle OAC Ror sin® sin(a-26) Let us write, or, r= 2R cos 0, 7= rcosOj7%rsin@j= 2Rcos*07+ Rsin20j” naaceea with respect to time. a a0 -> a ve 2R2cos 6(~sino) 22 dt O72 R 052020 or, v= 2n(=$° *) {sin 2077 om 207} ot, V= 2Re(sin20i—cos?0j) So, |v] or v= 2@R=0-4m/s. As @ is constant, v is also constant and w, = 2. 0, 2 2. So, we ee Gory. 407R = 032 m/s? Alternate : From the Fig. the angular velocity of the point A, with respect to centre of the circle C becomes _4(28) | 2{=48 a dt Thus we have the problem of finding the velocity and acceleration of a particle moving along a circle of radius R with constant angular velocity 2 w. )-20 = constant Hence v= 2wR and 2 2oR pooner Differentiating @ (t) with respect to time d an = o,= 2at (1) For fixed axis rotation, the speed of the point A: y veoR= 2a1R or R= > 2) 26 1.45 1.46 Differentiating with respect to time w= #. 2aR~ ©, (using 1) 2 But Wee x ee (using 2) R_ w/2at So, wa Vwlew? =V(v/t)+(2atvy = iVie4a7r The shell acquires a constant angular acceleration at the same time as it accelerates linearly. The two are related by (assuming both are constant) wee 1 2nn Where w= linear acceleration and =_angular acceleration Then, w= V2B2a0= V 24 (2any But v?= 2wi, hence finally - 2unv 7 Let us take the rotation axis as z-axis whose positive direction is associated with the positive direction of the cordinate @, the rotation angle, in accordance with the right-hand screw rule (Fig.) (a) Defferentiating p(t) with respect to time. Z $80 g-3517= 0, (t) and ao do, oo = Gee B= -601 Q) From (1) the solid comes to stop at At= t= ve The angular velocity w= a-3bt?, for 0-= dt 0 Similarly B= |B,|= 6b for all values of £. 1.47 1.48 27 Va/3b 6bt dt So, = = fat 2Vea /k 2Viy |e ep Hence < >= [ove 12 -f¥o 2 [2 - ays 0 We have @ = 0y- a9 = #2 Integratin this Eq. within its limit for (@) t -k [25 sf dt or, in 2m ket oO - @ Hence on Ge") ® (b) From the Eq., @ = wy + k@ and Eq. (1) or by differentiating Eq. (1) ake W= Wye Lt us choose the positive direction of z-axis (stationary rotation axis) along the vector B,, In accordance with the equation do, do, a” he 7d@ z or, do, = B,dp= Boospdg, Integrating this Eq. within its limit for ©, (9) * ° feo.- Bo} cos pap ° 0 ae or, oe Bo sin p Hence o,= + V2Bosing The plot «, (p) is shown in the Fig. It can be seen that as the angle @ grows, the vector @tirst increases, coinciding with the direction of the vector By (w, > 0), reaches the maximum at p= @/2, then starts decreasing and finally turns into zero at p = x. After that the body starts rotating in the opposite direction in a similar fashion (w, < 0). As a result, the body will oscillate about the position p= @/2 with an amplitude equal to 1/2. 29 1.51 Rotating disc moves along the x-axis, in plane motion in x~y plane. Plane motion of a solid can be imagined to be in pure rotation about a point (say /) at a certain instant known as instantaneous centre of rotation. The instantaneous axis whose positive sense is directed along @ of the solid and which passes through the point J, is known as instantaneous axis of rotation. Therefore the velocity vector of an arbitrary point (P) of the solid can be represented as : Sa y= Ox Tp () On the basis of Eq. (1) for the C. M. (C) of the disc ¥ wen Oxy 2 = oe According to the problem ¥;tti and Stt Fie. Gix-y plane, so to satisy the oe Eqn. (2) 7¢; is directed along (-j ). Hence point ~ x Tis at a distance r¢,= y, above the centre of QO the disc along y-axis. Using all these facts in Eq. (2), we get Ve Vom wy Or y= 7 Q) (a) From the angular kinematical equation @,= Og + Bt @ o= Bt On the other hand x= vt, (where x is the x coordinate of the C.M.) x or, t= a (5) From Eqs. (4) and (5), @ = fs Using this value of « in Eq. (3) we get jets = (hyperbola ) = mm = (b) As centre C moves with constant acceleration w, with zero initial velocity So, ede and v,= wt Therefore, y= wy = vixw Ye v2wx Hence oe oo 30 152 1.53 The plane motion of a solid can be imagined as the combination of translation of the C.M. and rotation about C.M. me So, we may write v= Voto = tox (1) and > > Wat Wot Wace a Wetw(-Ko)t(Bxrac) (2) Tac is the position of vector of A with respect to Ce In the problem v, = v= constant, and the rolling is without slipping i.e, vos v= OR, So, Wo = 0 and B= 0. Using these conditions in Eq. (2) 2 => => a v2 Was 07 (-M%c)= OR(-tye)= (ie) Here, age is the unit vector directed along Oe 2 a i and w, is directed along (-t,¢) or directed toward the centre of the Hence w, = wheel. (b) Let the centre of the wheel move toward right (positive x-axis) then for pure tolling on the rigid horizontal surface, wheel will have to rotate in clockwise sense. If w be the v angular velocity of the wheel then o = ee re Let the point A touches the horizontal surface at t= 0, further let us locate the point A att= 4, When it makes 8 = wf at the centre of the wheel. From Eqn. (1) Wyn Ver Ox re = = }+Rsin0(-i)] or, W= vit oR[coswt(-i)+sinotj ] = (v- cos wt) i+ vsinwtj” (as v= wR) So, vy = V (v-veos wt)? + (v sin wr)? = vV 2(1-cos wt) = 2 vsin (wt /2) Hence distance covered by the point A during T= 22x/w 2x/w a= fare ff av sinor2) a= 8R. 0 = vitw(-k)x [R cos 0 ( Let us fix the co-ordinate axis xyz as shown in the fig. As the ball rolls without slipping along the rigid surface so, on the basis of the solution of problem 1.52 : Mom Ve+Ox T= 0 Thus we 7 v,= oR and @tt(-k) as Vtti o 31 @,+Bx7rZ= 0 and y= BR and Btt ce) as Ww, ant7} A VetWR=Vs At the position corresponding to that of Fig., in accordance with the problem, B We= My 60 Vem WE \ oh and o- Rk and B= F (osing 1) Vp (a) Let us fix the co-ordinate system with the oe 7 attached with the rigid surface as shown in the Fig. As point O is the instantaneous centre of rotation of the ball at the moment shown in Fig. so, ¥o= 0, Now, he e+ Ox ige = Veit (HEX RG) = (ve+OR)E So, = 2veim 2wti (using 1) Similalry 7 = 95+ Bx Foo voit o(-F)xRG) 7 oe ry = v7 + OR (-J)= Veit Ve(-J) So, vg=V2 v,=VZ wt and Vp is at an angle 45° from both and j Fig.) (b) Wo Wow (- 73 )+ Bi oc A W #BR = 0? cide (Hoc) (using 1) where Mee is the unit vector along Toc \, 2 A B vj 50, wy Bat (using 2) and W, is 2 directed towards the centre of the ball Yee Now Wi, = Wo +0? (- 740) + Bc be 0 % = witw?R(-j)+B(-k)xXRj Ae 2\2 So, mye V awe = 2w 1+(5e] Similarly Wy = We + 0 (-F3c) + BX Te = wit wR (-1}+BCE)xRO) =\Y-z i+BR(-j) (using 1) 32 : (~ . t| i+ w(-j) (using 2) so, wa V + R 1.54 Let us draw the kinematical diagram of the rolling cylinder on the basis of the solutio: of problem 1.53. A Vaz2Vp Wet Br \ V7 We Br ve=VaVe As, an arbitrary point of the cylinder follows a curve, its normal acceleration and radius of curvature are related by the well known equation Ww, = > so, for point A, or, = 4r (because v.= wr, for pure rolling) Similarly for point B, Wainy ™ B W2v.¥ ‘B wr cos 45° = or, Ry= 2V2 me = 2v2r or 1.55 The angular velocity is a vector as infinitesimal rotation commute. Then the relative angular velocity of the body 1 with respect to the body 2 is clearly. y=, -a, as for relative linear velocity. The relative acceleration of 1 w.t.t. 2 is (#), 1.56 1.57 33 where S' is a frame corotating with the second body and S is a space fixed frame with origin coinciding with the point of intersection of the two axes, : di) (di) ut a le 7 oa ' ' Bs de Since S’ rotates with angular velocity w, . However oan | = 0 as the first body rotates with constant angular velocity in space, thus B= @) xa, Note that for any vector By the relation in space forced frame (k) and a frame (#) rotating with angular velocity @ is an dt Wehave i= atit bij” qd So, w= V (ai? + G2) , thus, ol, 30, = 7.81 rad/s Differentiating Eq. (1) with respect to time B= 18 ariany ) So, B= Var+(2 bie and Blea tos = 13 rad/s” ©) one oF, (ati br?7) (ait 2b) ) V (ay? + (br? V a? + (2b1y Putting the values of (a) and (b) atd'taking ¢= 10s, we get ae 17° (a) Let the axis of the cone (OC) rotates in anticlockwise sense with constant angular velocity &” and the cone itself about it’s own axis (OC) in clockwise sense with angular velocity @ (Fig.). Then the resultant angular velocity of the cone. FT +H a As the rolling is pure the magnitudes of the vectors @ and @, can be easily found from Fig. v * Reota’ @9= W/R 2) As & 1G, from Eq. (1) and (2) 34 o= Vo"+o3 ve 2 v\2 Vv (raz) +] ” Rosa ~ 23 7d/s (b) Vector of angular acceleration dv_4@+,) 7 7 (as @ = constant.) The vector @y which rotates about the OO’ axis with the angular velocity @, retains i magnitude. This increment in the time interval dt is equal to [d@y| = wg w' dt or in vector form d&g = (@ x Gy) dt. Thus B= Wx ay ¢ The magnitude of the vector Bis equal to B= ww (as D 135) v_v Y ana 2310d/s Bm ReotaR™ A The axis AB acquired the angular velocity oO = Bee (1) 0 Using the facts of the solution of 1.57, the SS angular velocity of the body W=BeE w= Var+o” = Vog + B20? = 0-6 rad/s Wo A And the angular acceleration. pe GS, SHH) ad doh Cth eee ye dy dw But Te x @y, and =e Bor So, B= (Bot x @)+ Bo As, BpL@ so, B= V (i Bot) + BS = By V1 + (wp) = 0-2 rad/s? 2 4.59 1.60 161 35 THE FUNDAMENTAL EQUATION OF DYNAMICS Let R be the constant upward thurst on the aerostat of mass m, coming down with a constant acceleration w. Applying Newton’s second law of motion for the aerostat in projection form F,= mw, mg-R= mw dd) Now, if Am be the mass, to be dumped, then using the Eq. F, = mw, R-(m- Am) g= (m-Am)w, Q) 2mw From Eqs. (1) and (2), we get, Am= a0 Let us write the fundamental equation of dynamics for all the three blocks in terms of projections, having taken the positive direction of x and y axes as shown in Fig; and using the fact that kinematical relation between the accelerations is such that the blocks move with same value of acceleration (say w) my &-T,= myw Q) T, - T, - kn, g=m,w (2) and T,-kmg= mw (6) The simultaneous solution of Eqs. (1), (2) and (3) yields, [mg - k (on, +m.) ] | —memem =F (1+k) my ant ng em, +, 28 mop As the block mg moves down with acceleration w, so in vector form pe (mto= kom + ma NE my +m, +m, Let us indicate the positive direction of x-axis along the incline (Fig.). Figures show the force diagram for the blocks. Let, R be the force of interaction between the bars and they are obviously sliding down with the same constant acceleration w. 36 1.62 Newton’s second law of motion in projection form along x-axis for the blocks gives : m, gsina — km, gcosa+R= mw 0) my g sina. -R— ky m,g cosa= mw Q) Solving Eqs. (1) and (2) simultaneously, we get km, + ham, : my +m, my my (k, ~ ke) g cos @ m, +m, (b) when the blocks just slide down the plane, w= 0, so from Eqn. (3) k m, + km, _ m, +m, or, (mm, + mz) sin o.= (ke, m, +k, m,) cos a _ &mtbhm) m,+m, Case 1. When the body is launched up : Let k be the coefficeint of friction, u the velocity of projection and I the distance traversed along the incline. Retarding force on the block = mg sina +kmg cos a and hence the retardation = gsina +kg cos o. Using the equation of particle kinematics along the incline, O= w-2(gsina+kgcosa)! w= gsina-gcos a ind R= @) gsin a - gcos a 0 Hence tana 2 2 1" FGsnaekgona) ® and O= u-(gsina+kgcosa)t or, us (gsin a +kg cos a) ¢ Q Using (2) in (1) [= A gsin +k gcos a)? @) Case (2). When the block comes downward, the net force on the body = mg sino - km g.cos & and hence its acceleration = g sin a ~ kg cos Let, ¢ be the time required then, I= 5 (gsin a - kg cos a)? a From Eggs. (3) and (4) £ | sina - kos a v2? sina+kcosa ButS= : (according to the question), Hence on solving we get 2 ke Gea punae 0-16 ne + 163 1.64 165 37 At the initial moment, obviously the tension in the thread connecting m, and m, equals the weight of m,. (@) For the block m, to come down or the block m, to go up, the conditions is m,g-Tz0 and T-m,gsina-fre0 where T is tension and f, is friction which in the limiting case equals km,g cosa. Then or mg-mysina>km,gcosa or 2, (k cos 0 + sin a) m (b) Similarly in the case m, gsina—m)§> frig or, m, gsin a - mg > km, g cos a or, 7 & (sin o.- kas 0) m™ (©) For this case, neither kind of motion is possible, and fr need not be limiting. Hence, (k eos 0 + sin a) > 2 > (sin a - cos 0) 1 From the conditions, obtained in the previous problem, first we will check whether the mass m, goes up or down. Here, m,/m, = > sin a + kcos a, (substituting the values). Hence the mass m, will come down with an acceleration (say w). From the free body diagram of previous problem, m,-g-T= mw @) and T-m,gsina-km,gcosa= mw (2) Adding (1) and (2), we get, m, gm, g sin a - km, gcos a= (m, +m) w (m,/m, -sina-keosa)g _ (y-sina-kcosa)g (1 + m,/m,) 1+y Substituting all the values, w= 0-048 g ~0-05 ¢ ‘As m, moves down with acceleration of magnitude w= 0.05 g > 0, thus in vector form acceleration of m: w oa M-sina-kes a)” pos ¥, isa 0.05 8° Let us write the Newton’s second law in projection form along positive x-axis for the plank and the bar fr= mw,, fr= mw, qa) 1.66 1.67 At the initial moment, fr represents the static friction, and as the force F grows so does the friction force fr, but up to it’s limiting value ie. fr= frgmaxy= KN = km, g. fr. F Unless this valuc is reached, both bodies moves Fp as a single body with equal acceleration. But as soon as the force fr reaches the limit, the 7. bar starts sliding over the plank i.e. w,2 W,. Substituting here the values of w, and w, taken from Eq, (1) and taking into account that f= km, g,we obtain, (at - kan, g)/m, = , were the sign "=" corresponds to the moment m, t= ty (say) k 1 If tf, then , - lame (constant). and ‘W, = (at - km, g)/m, On this basis w, (t) and w, (#), plots are as shown in the figure of answersheet. Let us designate the x-axis (Fig.) and apply F, = mw, for body A : mgsina-kmgcosa= mw or, w= gsna-kgcosa Now, from kinematical equation : Isec a= 0+ (1/2) wt? or, t= V2Tsec a/(ein a — F008 a) 7 = V21] (sin2 0/2 - keos a) g Ly (using Eq, (1)). (Mp2 tex) for bain» cae: 0 A t—_ ie. 20820 > kcosasina = 0 or, tn2a= -Lora= 49° k and putting the values of a, k and / in Eq. (2) We get fai, = 1s. Let us fix the x-y co-ordinate system to the wedge, taking the x-axis up, along the incline and the y - axis perpendicular to it (Fig.). 39 Now, we draw the free body diagram for the bar. Let us apply Newton’s second law in projection form along x and y axis for the bar : Tcos B-mgsina-fr= 0 (1) Tsin B+N-mgcosa= 0 or, N= mgcosa-TsinB @ But f= kN and using (2) in (1), we get T= mg sina + kmg cos a/(cos B + ksin B) @) For Tru, the value of (cos B + ksin B) should be maximum d(cos B+ksin So, dees Be ksin ) =0 or tnpek Putting this value of B in Eq. (3) we get, _ —meg(sina+kcosa) __ mg (sina +k cos a) n r 1/Viek? +h /Vi+ Viek? mat First of all let us draw the free body diagram for the small body of mass m and indicate x-axis along the horizontal plane and y — axis, perpendicular to it, as shown in the figure. Let the block breaks off the plane at t= % ic. N= 0 So, N= mg-atsina= 0 or, tg= oR Q) From F, = mw,, for the body under investigation : md y/dt = atcos a ; Integrating within the limits for v(t) mf a, acosa fat (using Eq, 1) 0 0 So, ve s. ose? Q) Integrating, Eqn. (2) for s (t) 3 acosat fo amas: @) Using the value of t= fy from Eq. (1), into Eqs. (2) and (3) mg cosa mi g° cosa ve and s= 2asin’a 6a’ sin? a 40 1.69 1.70 L7L Newton’s second law of motion in projection form, along horizontal or x-axis i.e. F,= mw, gives. F cos (as) = my (as a= as) or, Fcos (as) ds = mvdv Integrating, over the limits for v (s) E ¥ se coon de a or ve Vv ma = V2gsina/3a (using F= ™) which is the sought relationship. From the Newton’s second law in projection from : For the bar, T-2kmg = (2m)w @ For the motor, T- kang = mw’ (2) Now, from the equation of kinematics in the frame of bar or motor : l= Sows wys @) From (1), (2) and (3) we get on eliminating T and w t= V2i/(kg+3w) Let us write Newton’s second law in vector from F'= mw, for both the blocks (jin the frame of ground). T+ mg mW @ Ts m3 maw @ These two equations contain three unknown quantities #,, W, and T. The third equation is provided by the kinematic relationship between the accelerations : a ee Wis WotW , Wye WH 3) where Wis th acceleration of the mass m, with respect to the pulley or elevator car. Summing up termwise the left hand and the right-hand sides of these kinematical equations, we get 172 41 Wr i= 2 @ The simultancous solution of Eqs-(1), (2) and (4) yields >, = ae _ (m=) F4 2m, Wy a my, My Using this result in Eq. (3), we get, cy _ My-M, > —», zr 2mm, _, we mam, 27) and Te meme =) m— > —>, Using the results in Eq. (3) we get w" = 2-2 (gift) m,+m, (b) obviously the force exerted by the pulley on the celing of the car > = 4mm. => Fe -2T= 2 gm) at ia: Note : one could also solve this problem in the frame of elevator car. Let us write Newton’s second law for both, bar 1 and body 2 in terms of projection having taken the positive direction of x, and x, as shown in the figure and assuming that body 2 starts sliding, say, upward along the incline T, -m, gsina= m, Q) ig N m,§-T,= m,w Q) tT For the pulley, moving in vertical direction Ti from the equation F, = mw, 2T)-T,= (m,)w,= 0 x mg. bh | (as mass of the pulley m,= 0) 2 ¥%2 or T= 07; @) As the length of the threads are constant, the mg. kinematical relationship of accelerations becomes w= 2m (4) Simultaneous solutions of all these equations yields : m . 2g|2-7-sina : 2g(2 in a) (Ba) a we ‘As > 1, w is directed vertically downward, and hence in vector form —» 22°2y-sina) we S8eN~ sino) 4n+1 42 1.73 1.74 Let us write Newton’s second law for masses m, and m, and moving pully in vertical direction along positive x - axis (Fig.) : m,g-T= mw, qa) i. —x m,g-T= mW, ) T,-2T= 0(asm= 0) or T,-2T @) Again using Newton’s second law in projection form for mass my along positive x, direction (ig.), we get T,= my @ The kinematical relationship between the accelerations of masses gives in terms of projection on the x - axis Wy t Wo, = 2 Wo (5S) Simultaneous solution of the obtained five equations yields : vo, = EAM Ma + my (my ~ my) 18 7 4m, m, + mo (m, +m) In vector form ce, Am rig + 1g (rm, - ) 18” 1 "Aimy mn, + tm (mm, + m3) As the thread is not tied with m, so if there Were no friction between the thread and the ball m, the tension in the thread would be zero and as a result both bodies will have free fall motion. Obviously in the given problem it is the friction force exerted by the ball on the thread, which becomes the tension in the thread. From the condition or language of the problem w,,>w,, and as both are directed downward so, relative acceleration of M = wy,-w,, and is directed downward, Kinematical equation for the ball in the frame of rod in projection form along upward direction gives : PS y= Wy) @ Newton’s second law in projection form along Tz f vertically down direction for both, rod and ball ly gives, Mg -fr= Mwy Q) fr gfe mig @ wy ee Multiplying Eq. (2) by m and Eq. (3) by M ml and then subtracting Eq. (3) from (2) and after ¢ using Eq. (1) we get y 21Mm Orem a "4 LIS 1.76 43 Suppose, the ball goes up with accleration w, and the rod comes down with the acceleration w. As the length of the thread is constant, 2, = w, q@) From Newton’s second law in projection form along vertically upward for the ball and vertically downward for the rod respectively gives, T-mg~ mw Q and Mg-T'= Mw, @) but T=2T (because pulley is massless) (4) From Eqs. (1), (2), (3) and (4) _ OM=m)g_ (=n, ae We STEM acrid (in upward direction) and w)= are (downwards) From kinematical ee in projection form, we get I. Fm +) 1? as, w, and Ww, are in the opposite direction. Putting the values of w, and w,, the sought time becomes = V2I@+ O13 Q-We= 14s Using Newton’s second law in projection form along x - axis for the body 1 and along negative x ~ axis for the body 2 respectively, we get m,g-T,= my @) T,~m,g = mw, Q) For the pulley lowering in downward direction from Newton’s law along x axis, -2T) = 0 (as pulley is mass less) on, eee (@) As the length of the thread is constant so, wpe 2m, ® The simultaneous solution of above equations a nye LRP UO=D ag Th ny ©) Obviously during the time interval in which the ay 1 comes to the horizontal floor covering the distance h, the body 2 moves upward the distance 2h. At the moment when the body 2 is at the height 2h from the floor its velocity is given by the expression : B= 25, (2) = 2[ 20-28), Shn=2) n+4 n+4 After the body m, touches the floor the thread becomes slack or the tension in the thread zero, thus as a result body 2 is only under gravity for it’s subsequent motion. 44 LH 1.78 Owing to the velocity v, at that moment or at the height 2h from the floor, the body 2 further goes up under gravity by the distance, 2g n+4 Thus the sought maximum height attained by the body 2 : He 2heh = 2h 0-2 Snk ™+4) n+4 Let us draw free body diagram of each body, ic. of rod A and of wedge B and also draw the kinemetical diagram for accelerations, after analysing the directions of motion of A and B. Kinematical relationship of accelarations is : Wa taa= a) Let us write Newton’s second law for both bodies in terms of projections having taken positive directions of y and x axes as shown in the figure. m,8-Ncos a= mW, 2) and Nsina= m,w, @) Simultaneous solution of (1), (2) and (3) yields : m,gsina Wy es Band m, Sin & + mg COLaCOSa (1 + Ncot” a) i “tana” (ana+1 cota) Ws — Wap Wy = Wg Note : We may also solve this problem using conservation of mechanical energy instead of Newton’s second law. Let us draw free body diagram of each body and fix the coordinate system, as shown in the figure. After analysing the motion of M and m on the basis of force diagrams, let us draw the kinematical diagram for accelerations (Fig.). As the length of threads are constant so, say = dsy and as Vry and vy, do not change their directions that why [ae | = [Pe |= w (say) and Wr tty and Wh tt Vy 1.79 = Wn rT? ro fe N Nal Ny Wn Tv > Winn T ™ =x Ft 7 Wm AS Why = Wray + Wy so, from the triangle law of vector addition Wy = V2 w q@) From the Eq. F,= mw, , for the wedge and block : T-N= Mw, Q@) and N= mw @) Now, from the Eq. F,= mw,, for the block mg -T-kN= mw (4) Simultaneous solution of Eqs. (2), (3) and (4) yields : we —_—™s __ = coe (en +2m+M)” (k+2+M/m) Hence using Eq. (1) oo eR Vou mm” (24k+M/m) Bodies 1 and 2 will remain at rest with repect to bar A fOr Wig SWS Wyazy WHETE Wyn i the sought minimum acceleration of the bar. Beyond these limits there will be a relative motion between bar and the bodies. For 0 < ws w,,;., the tendency of body 1 in relation to the bar A is to move towards right and is in the opposite sense for W= Wa, On the basis of above argument the static friction on 2 by A is directed upward and on 1 by A is directed towards left for the purpose of calculating W,,. Let us write Newton’s second law for bodies 1 and 2 in terms of projection along positive x-axis (Fig.). T-fry= mw ot, fry= T-mw (1) N,= mw (2) As body 2 has no acceleration in vertical direction, so f= mg-T (3) From (1) and (3) (fr, + fra) = m(g-w) (4) But fr, + fry sk (N, +N) or fry + fry sk (mg + mw) () 46 1.80 1.81 From (4) and (5) m(g-w) k ‘max = Let us write Newton’s second law for both bodies in projection form along positive Yq and x, axes as shown in the Fig. 2G) 22] Gc.) | ale | or, m,g.cos -N= m,w, sino qd) and Nsina = mw, Q) Solving (1) and (2), we get my g sin a. cos & sin a. cos a wy» 2 = aaa m, + m,sin? 0 (m,/m, ) + sin* a Wi 47 1.82 To analyse the kinematic relations between the bodies, sketch the force diagram of each 1.83 body as shown in the figure. On the basis of force diagram, it is obvious that the wedge M will move towards right and the block will move down along the wedge. As the length of the thread is constant, the distance travelled by the block on the wedge must be equal to the distance travelled by the wedge on the floor. Hence d 5,4 = dS AS Vey and Vy, do not change their directions and acceleration that’s why Wy y tt Vey, and Wy tt Vj and Way = Wy = (say) and accordingly the diagram of kinematical dependence is shown in figure. : Wm AS Wy = Wry + Wy, 80 from triangle law of vector addition. Wy Vig + Wey 2 Way Wy 00s 2 = wV2(1 — cos a) (1) From F, = mw, , (for the wedge), T= Tcosa+Nsina= Mw Q) For the bar m let us fix (x -y) coordinate system in the frame of ground Newton’s law in projection form along x and y axes (Fig.) gives mg sina - T= mw, ()= m [ ml (xy uc] =m [Wat 4 008 ( - a] = mw (2 — cosa) @) mgcosa—-N= mWay)= mm Wmaciyy * Macy) | m(O+wsina] (4) Solving the above Eqs. simultaneously, we get o mgsina M+2m(1-cosa) Note : We can study the motion of the block m in the frame of wedge also, alternately we may solve this problem using conservation of mechanical energy. Let us sketch the diagram for the motion of the particle of mass m along the circle of radius R and indicate x and y axis, as shown in the figure. - (a) For the particle, change in momentum A p= my (-i)-mv(j) so, [4 P| |= V2 my and time taken in describing quarter of the circle, 48 1.84 1.85 aR Are 38 a 2 Hence, = LAPL, V2mv _ 2V2 mv? > 4 At mR/2v aR Vp => () In this case VE P= 0 and py= mw,t(-i), so |Ap|=mwt Hence, || = lazl. . While moving in a loop, normal reaction exerted by the flyer on the loop at different points and uncompensated weight if any contribute to the weight of flyer at those points. (a) When the aircraft is at the lowermost point, Newton’s second law of motion in projection form F, = mw, gives my? N-mg= one 2 or, N= mg +2 = 2:09 KN (b) When it is at the upper most point, again from F, = mw, we get emg ” my? N" = Mm g= 0-7KN (c) When the aircraft is at the middle point of the loop, again from F,,= mw, 2 ove N' R 1-4kN The uncompensated weight is mg. Thus effective weight = VN" +m" g” =1-56kN acts obliquely. Let us depict the forces acting on the small sphere m, (at an arbitrary position when the thread makes an angle @ from the vertical) and write equation F. = mwvia Projection on nen the unit vectors i, and u,. From F,= mw,, we have mg sin Om & vay _vdv = mee ds ~ "1(-d0) (as vertical is refrence line of angular position) 49 or vdve —glsind dd Integrating both the sides : v 8 dv= -glf sin0d0 {vw elf sin 2 or, 7 =glcos® y2 Hence “= 2gcos = w, a) ar (Eq. (1) can be easily obtained by the | conservation of mechanical energy). \ From F,= mw, may? i T-mgcoso=™?, A o Using (1) we have mq T= 3 mgcos0 2) Again from the Eq. F,= mw,: mg sinO=mw, or w,= gsin® @) Hence w= Vw2+w? = V(gsin 0)? +(2g cos 0)” (using 1 and 3) = gV1+3 cos? (b) Vertical component of velocity, v, = v sin @ So, v2 = v?sin?@ = 2g 1cos @ sin? (using 1) . 2 d(cos sin? 0) For maximum VOt epee 1 which yields cos O= Therefore from (2) T= 3mg Fe = VE mg (©) We have W= w, ii, +W, ll, thus W, = Way) + Way) But in accordance with the problem w, = 0 Wiyy + Hayy) = 0 or, gsin 0 sin 0 + 2g cos 70 (- cos ®) = 0 1 or, cosO= 7 oF, O= 547° 50 1.86 1.87 The ball bas only normal acceleration at the lowest position and only tangential acceleration at any of the extreme position. Let v be the speed of the ball at its lowest position and / be the length of the thread, then according to the problem - gsina (1) where a is the maximum deflection angle From Newton’s law in projection form : F,= mw, mg sin 0 = mv 140 or, -glsinOd0= vdv On integrating both the sides within their limits. a 0 ~alf sinodo= f vdv 0 ’ or, v? = 2g! (1 - cos a) 2) Note : Eq. (2) can easily be obtained by the conservation of mechanical energy of the ball in the uniform field of gravity. From Eqs. (1) and (2) with @ = 2gl (1 - cos a) = Ig cos a or, cos a= 2 so, a= 53° Let us depict the forces acting on the body A, (which are the force of gravity mg’and the normal reaction N ) and write equation F = mw via projection on the unit vectors u, and 4, (Fig.) From F,= mw, : dv mg sin 0 = ™E oe ee Pia RaO or, gRsin0d0= vdv Integrating both side for obtaining v (0) e v N J gRsinodo~ fray 0 0 R i, or, \\v? = 2.gR (1 - cos 8) R Le & wd From F,= mw, , De mg cos 0- N= me mg, Q) At the moment the body loses contact with the surface, N= 0 and therefore the Eq. (2) becomes v= gRcos@ QB) 1.88 1.90 51 where vand @ correspond to the moment when the body loses contact with the surface. Solving Eqs. (1) and (3) we obtain cos 6 = 2 of, 8= cos”! (2/3) and v= V2gR73. At first draw the free body diagram of the device as, shown. The forces, acting on the sleeve are it’s weight, acting vertically downward, spring force, along the length of the spring and normal reaction by the rod, perpendicular to its length. Let F be the spring force, and Al be the clongation. From, F, = mu, Nsin0+F cos0= mw*r a where rcos 0 = (I, + AD. Similarly from F,= mw, Ncoos@-FsinO= 0 or, N=Fsin0/cos 0 (2) From (1) and (2) F (sin 0/cos 8) -sin@+F cos 0 = mur = mo? ({, + Al)/cos 8 On putting F= x Al, x Asin? 6+ Kk Alcos?0 = mw? (Iy+AD) on solving, we get, a K-mo (K/mw?-1) and it is independent of the direction of rotation. According to the question, the cyclist moves along the circular path and the centripetal force is provided by the frictional force. Thus from the equation F, = mw, Al= mo? 2 or {igen > or = ky(r-P/R)g q) a(r For vagy) We should have 4-0 or, 1-2. 0, so r= R/2 Hence Ygg= $V Fe ah As initial velocity is zero thus v= ws Q@) As w,>0 the speed of the car increases with time or distance. Till the moment, sliding Starts, the static friction provides the required centripetal acceleration to the car. Thus fr= mw, but frs kmg 52 1.91 1,92 So, ws Reg or, Weta? or, vs (Pg -w)R Hence Vaan VE = WR so, from Eqn. (1), the sought distance s = foot = = 6m. Since the car follows a curve, so the maximum velocity at which it can ride without sliding at the point of minimum radius of curvature is the sought velocity and obviously in this case the static friction between the car and the road is limiting. Hence from the equation F, = mw kg = ae or vs VERE so Vasag = VE Rain 8 + (1) ‘We know that, radius of curvature for a curve at any point (x, y) is given as, [1+ Gy/aey 7 Re enlae For the given curve, 8 og (*) and TY m ~8 ein = 7 ae a and ae ne Substituting this value in (2) we get, [1 + (a2/a2) cos? (x/a) 7 nS 2) (a/a’) sin (x/a) For the minimum R, x. = 2 and therefore, corresponding radius of curvature 2 a Rag ® Hence from (1) and (2) Venax = Vg /a The sought tensile stress acts on each clement of the chain. Hence divide the chain into small, similar elements so that each element may be assumed as a particle. We consider one such element of mass dm, which subtends angle dc at the centre. The chain moves along a circle of known radius R with a known angular speed w and certain forces act on it, We have to find one of these forces. From Newton’s second law in projection form, F, = mw, we get 2Tsin (da./2) - dN cos 0 = dmw”R and from F, = mw, we get dNsin® = gdm Then putting dm = mdo/2x and sin (da/2) = da/2 and solving, we get, m (wR +g cot 0) Ta Mo Regt) 2n 53 Ty " dmg : 1,93 Let, us consider a small element of the thread and draw free body diagram for this element. (@) Applying Newton’s second law of motion in projection form, F, = mw, for this element, (T+ dT) sin (d 0/2) + Tsin (d0/2)-dN= dmw’R= 0 or, 27 sin (d 0/2) = AN, [negelecting the term(dT sin d 8/2) ] _d0_ do or, Td0- AN, as sin = 5 0) Also, dfr= kdN= (T+dT)-T= dT T Q) From Eqs. (1) and (2), kTd0= aT or “Fa kao In this case Q= x so, nee 3 or, or, na x @) 7 So, ke Zin em Zinn m. (b) When —>= n, which is greater than my the blocks will move with same value of 1 acceleration. (say w) and clearly m, moves downward. From Newton’s second law in projection form (downward for m, and upward for m,) we get : m,g-T,= m,w (4) and T,-m,g= mw 6) 54 1.94 1.95 1.96 197 t Also 7,7 7 © Simultaneous solution of Eqs. (4), (5) and (6) yields : _ m= Nom )8 _ (=) my (m,+n9m) — (nem) ® (Sm " The force with which the cylinder wall acts on the particle will provide centripetal force necessary for the motion of the particle, and since there is no acceleration acting in the horizontal direction, horizontal component of the velocity will remain constant througout Ve) the motion. So Vg Vp COS 0 Using, F, = mw,, for the particle of mass m, mvz mv,cos"a N= eee re which is the required normal force. Obviously the radius vector describing the position of the particle relative to the origin of coordinate is r= xityj= asinwtit+bcoswtj Differentiating twice with respect the time : ao We © F = ~ 08 (asin w i+ boos wr j3~ -o 7 @) t Thus F= mif= -morr” @) Wehave p= f Fa ' = [mzacm mz a ° > : : : : 2(%8) (b) Using the solution of problem 1.28 (6), the total time of motion, t= - > Hence using t=tin (1) [4p] = mgt nat aes = -2m(vog)/g (19° 8 is -ve) From the equation of the given time dependence force F'= a t(t-1) at t=, the force vanishes, (@) Thus a= p= f Fa o 55 > we or, Pf diene 0 > ae but (b) Again from the equation F= mix" at (x1) =m or, GUtv-t?)dt= md Integrating within the limits for v%), or, Thus Hence distance covered during the time interval t= +, . se fva 198 We have F = Fysin wt or m4¥ 6 F,sin ot or mdv™ F,sin wt de On integrating, = -F . A inly = —* cos wt + C, (where C is integration constant) = Fo When t= 0, v=0, so C= ——~ mo a», ~Fo Ce ae eee nee See mo mo Fo As | cos wt < 1 so, v= —*-(1-cos of) mo 56 1.99 1.100 ' Thus ss fr at 0 Fit Fosinwt 0 mo mo ma (ot - sin wt ) (Figure in the answer sheet). oS According to the problem, the force acting on the particle of mass m is, F = F, cos wt > _ F So, mY. Fcosut or dv 2 cosatdt ad m Integrating, within the limits. Fy + F far 70 f cos art or v= —“sin wt m mo oO oO It is clear from equation (1), that after starting at t= 0, the particle comes to rest fro the first time at r= —. @ Fi 1 - fo fe x ‘rom Eq. (1), v= |v|= mmo SOF for ts (2) Thus during the time interval ¢ = x/w, the sought distance p vo so 22 f sinavare = my mo From Eq. (1) Fo “ex" Go | sin we | <1 > re @) From the problem F= -7 so m= -1v dv =>_ = Thus mae -w [as dvt}v] or, ite vom On integrating Inv=-7 64 But at t= 0, v=v, so, C= Invy or, -2t of, v= _ m Thus for tao, v= 0 dv =r (b) We have m= -rv so dv= —"ds 1.101 87 Integrating within the given limits to obtain v(s): : or, fave -2fas or vevy- (a) % 0 mvo Thus for v= 0, 5 Sia = (© Letwehave py or Mame v vom J : or, +. tf m o 0 So a =m In(1/y) _ ming r r Now, average velocity over this time interval, Man r n - fineka vdt % y(n-1) = —_—-—___ fa Zinn nny r According to the problem dv 2 dv mae -kv* ot, more kde Integrating, withing the limits, . ' ae m (%o-¥) Se fa on 1 BS © i To fine the value of k, rewrite Oo we mas kv* or, Y a ds On integrating ’ So, Q) Putting the value of k from (2) in (1), we get i. A(v-v) vin “2 ey, 58 1.102 1.103 From Newton’s second law for the bar in projection from, F,= mw, along x direction we get mg sin a - kang cos a = mw So or, v Se = gsina-axrgcos.a, (as k= ax), or, vdv= (gsin a -axg cos a) dr or, Jf vdv= af (sina -xc0s a) de 0 0 2 2 - x So, “= g(sinax-*-acosa) 4 778 2 qa) N fr From (1) v= 0 at either x20, orx= 7 tana a As the motion of the bar is unidirectional it stops after going through a distance of % _ < e 2 tana. a From (1), f0F Vass (sin ox - acosa)« 0, which yields x= Fauna Hence, the maximum velocity will be at the distance, x = tan a/a Putting this value of x in (1) the maximum velocity, gsin a tan a Vinax ‘max a Since, the applied force is proportional to the time and the frictional force also exists, the motion does not start just after applying the force. The body starts its motion when F equals the limiting friction. Let the motion start after time f) , then ka F = aty=kmg or, t= a So, for t= s fy, the body remains at rest and for £ > f obviously mdv ane a(t-t) or, mdv= a(t-t)dt Integrating, and noting v = Oat ¢ = fy, we have for t> ty y ‘ a 2 mdv=a | (t-t))dt or v= Im (= fo) 0 in Thus (1-1) dt= Sq lt) 1.104 1.105 While going upward, from Newton’s second law in vertical direction : vdv kv? (+) At the maximum height A, the speed v = 0, so 0 A Scitm S* mB mgs kv?) ot = gs ds Integrating and solving, we get, @ mg When the body falls downward, the net force acting on the body in downward direction equals (mg- kv"), Hence net acceleration, in downward direction, according to second law of motion vdv |? vdv as aE a ee _ = A Thus f4-Je g-kv*/m o 0 Integrating and putting the value of h from (1), we get, vie vo/ vi +kve/mg- Let us fix x - y co-ordinate system to the given plane, taking x-axis in the direction along which the force vector was oriented at the moment f= 0, then the fundamental equation of dynamics expressed via the projection on x and y-axes gives, FP dv, coswt= me (4) d and Fsinot= mot ® dt (a) Using the condition (0) = 0, we obtain v, = = sinwt Q) and EF v= 5 (1- cos wr) (4) Hence, vev ve ty = ( . ) mo “(?)| 60 1.106 1.107 (b) It is seen from this that the velocity v turns into zero after the time interval At, which can be found from the relation, of. x, Consequentely, the sought distance, is ar sa f va a ‘ mo vdt Average velocity, = fae 2Wo So, -f2z sin($*)/ (220) 0 The acceleration of the disc along the plane is determined by the projection of the force of gravity on this plane F,= mg sina and the friction force fr = kmg cos a.. In our case k= tana and therefore amo fr= F,=mg sina Let us find the projection of the acceleration on the derection of the tangent to the trajectory and on the x-axis : mw,= F,cos @ - fr = mg sin a( cos p-1) mw, = F,- frcos p= mg sin 0. (1- cos p) It is seen fromthis that w, = — w,, which means that the velocity v and its projection v, differ only by a constant value C which does not change with time, icc. ve y+, where v,= v.cos p. The constant C is found from the initial condition v= vp, whence C= since p = 7 initially. Finally we obtain v= vy/(1+cos@). In the cource of time p-+ 0 and v—> v9/2. (Motion then is unaccelerated.) Let us consider an element of Jength ds at an angle @ from the vertical diameter. As the speed of this element is zero at initial instant of time, it’s centripetal acceleration is zero, and hence, dN - dds cos p= 0, where 2 is the linear mass density of the chain Let T and T+dT be the tension at the upper and the lower ends of ds. we have from, F, = mw, (T+ dT)+hdsgsing-T= ddsw, or, aT +hRd gg sing = dds, 1.108 61 If we sum the above equation for all elements, the term f dT = 0 because there is no tension at the free ends, so VR rar f singdo= dw, f ds=d1w, 0 Hence w,= S(1-eoo5] As w, =a at initial moment =], |= FE (1 ~ cos So, w=|w,| T [17 8 R In the problem, we require the velocity of the body, realtive to the sphere, which itself moves with an acceleration Wy in horizontal direction (say towards left). Hence it is advisible to solve the problem in the frame of sphere (non-inertial frame). At an arbitary moment, when the body is at an angle @ with the vertical, we sketch the force diagram for the body and write the second law of motion in projection form F,= mw, mv 2 of, mg cos 0 ~ N ~ mvy sin 0 = 7 — 0) At the break off point, N= 0, @= Oy and let v= vqso the Eq, (1) becomes, 2 z 8 COs Oy — Wy Sin O e) From, F,= mw, i6 nipco@ = meee mg sin 0 — mw, cos 0 = m~* a or, vdv= R(gsin 0 + w,cos0)d0 % % Integrating, fv dv =fr (g sind + w, cos0) d 0 0 ° 2 y aR B(1 - cosOy) + wy sin Oy @) Note that the Eq. (3) can also be obtained by the work-energy theorem A= AT (in the frame of sphere) therefore, mgR (1 - cos Op) + mwa R sin y= 3 mvz [here mw, R sin Qq is the work done by the pseudoforce (- mw,)] 2 0 . cn BR™ (1 ~€05 Op) + wo sin Op 62 1.109 1.110 Solving Eqs. (2) and (3) we get, 24+kV5 + 9k? Wo vo = V2gR73 and 0, =cos~!| ———-——— |, where k= — q ‘ 3(1+k?) 8 Hence % lees 17° This is not central force problem unless the path is a circle about the said point. Rather here F, (tangential force) vanishes. Thus equation of motion becomes, ¥,= Vo constant 2 myo A d, —s= fe - and, pe de We can consider the latter equation as the equilibrium under two forces. When the motion is perturbed, we write r= ry +x and the net force acting on the particle is, 2 2 i my -—4—4 On ee (eee (7rg+x)" Mott rg % % To mv =-—z (1-n)x To 2 my; This is opposite to the displacement x, if n<1- ( is an outward directed centrifugul force while aed is thé inward directed external force). r There are two forces on the sleeve, the weight F, and the centrifugal force F,. We resolve both forces into tangential and normal component then the net downward tangential force on the sleeve is, 2 mg sinO 1-*Fesea} This vanishes for 0.= 0 and for 0 @= 0) = cos~!(—£—), which is real if oR 2 2 MW?RSinOCosO wo R>g. If o*Rg, 1- SF coo isnegative forsmall @ near 0 = 0 and 0 = 0 is then unstable. However 0 = , is stable because the force tends to bring the sleeve near the equilibrium position @ = Qy, If wR = g, the two positions coincide and becomes a stable equilibrium point. 1111 1112 1113 63 Define the axes as shown with z along the local vertical, x due east and y due north. (We assume we are in the northern hemisphere). Then the Coriolis force has the components. FE, = -2m@xv)} = 2me | v, cos® - v, sin0) i= v, cos j + v, cos k] = 2m (v, cos0 — v, sind) 7” since v, is small when the direction in which the gun is fired is due north. Thus the equation of motion (neglecting centrifugal forces) are X= 2mo (v, sing - v, cos), y= 0 and Z = -g y-North Integrating we get y= v (constant), z= - gt v z-vertical and X= 2 vsing t+ wg t? cosp East Finally, x= ov? sing + 3 at? cose Now v >> gt in the present case. so, 2 x= ovsing(*) = wsin ci Ply ey ~ 7cm (to the cast). The disc exerts three forces which are mutually perpendicular. They are the reaction of the weight, mg, vertically upward, the Coriolis force 2mv’ w perpendicular to the plane of the vertical and along the diameter, and mor outward along the diameter. The resultant force is, Fa mVe+o'rs (vo The sleeve is free to slide along the rod AB. Thus only the centrifugal force acts on it. The equation is, my = mor where ved. pe yp@e Aly Butve vie $(2”) s0, ive $0? ? + constant or, ve heat? vg being the initial velocity when r= 0. The Coriolis force is then, amo Vigra = 2a? VIE = 2:83 N on putting the values. 64 1.114 1115 The disc OBAC is rotating with angular o! velocity about the axis OO! passing through the edge point O. The equation of motion in w rotating frame is, mitt = Fs mo? R's Ins’ xd FoF, 8 where F,, is the resultant inertial forc. (pseudo force) which is the vector sum of centrifugal 0 A and Coriolis forces. (a) AtA,F,, vanishes. Thus 0 = -2mw?Rn+2mv' on where 7 is the inward drawn unit vector to the centre from the point in question, here A. Thus, v= oR 2 yi? 2 so, Wa eo R. = i at (b) AtB F,,= ma OC +mo* BC its magnitude is mo? VaR? — r , where r= OB. The equation of motion in the rotating coordinate system is, = > mi" = F+ mw? R+ 2m ("x 3) Now, v= RO@+Rsind pe and = W = w'cosOe,-w' sin Oa . oo 6 Fee ® 0 RO Rsindp @cos 8 -wsin® 0 = (oR sin0 @) + wR sin 0 cos 0 @ - WR O cos 02, Now on the sphere, a 2m Ve (-RO-Rsin? 0g?) a + (RO -Rsin 6 cos 0g") e + (Rsin 8p" + 2R cos 6 6) e Thus the equation of motion are, m(-R @-Rsin’ 6 g”) = N- mg cos 0 + mw” R sin’ 6 + 2mw R sin? 0p m(R@ -Rsin @ cos @ g”) = mg sin 0 + mw” R sin 0 cos 8 + 2mw R sin @ cos 0 @ m(Rsin 0p +2R cos 06) = -2mwR Ocos @ From fhe third equation, we get, p= -@ A result that is easy to understant by considering the motion in non-rotating frame. The eliminating 9 we get, i mR 6? = mg cos @-N mRO = mgsin® Integrating the last equation, imR Pm mg (1 - cos 8) 1.116 LUT 65 Hence Nx (3-208 8) mg So the body must fly off for @= 0, = cos~?2, exactly as if the sphere were nonrotating. > Now, at this point F.,= centrifugal force = mc” R sin 0) = v2 mor R Fy = Vo RO cos? 0 + (wR) sin’ @ x 2m = V2 2 R24 = me? aces V2 WRF +0) Rbk x 2m = 5 mos RY s+ (a) When the train is moving along a meridian only the Coriolis force has a lateral component and its magnitude (see the previous problem) is, 2m wv cos 8 = 2m sind (Here we have put R 6 v) 34, 2%, $4000 , v5 So, Figurat ™ 2% 2000 x 10° x SARE x Sx = 3-77KN, (we write 2 for the latitude) (b) The resultant of the inertial forces acting I> 7 on the train is, F=-2moROcos0e CPN Feot + (ma? R sin 0 cos 8 + 2m w R sin 8 cos ©) 2 (>) + (mo? R sin? 0 + 2m w R sin? 09 )e, This vanishes if O= 0, @= -Fo > > 1 A 1 Thus VE Wey: Yo™ -7oRsinO= ->0R cosh (We write X for the latitude here) Thus the train a move from the east to west along the 60" parallel with a speed, 1 -4 6 | SoReosh= 4x22 x10 x 6:37 x 10° = 115-8 m/s = 417 km/hr We go to the equation given in 1.111. Here v,= 0 so we can take y= 0, thus we get for the motion in the xz plane. x = ~2wv,cos 8 and e-g : 1 2 Integrating, z= -58 = wgcos gt? i a So x= 3wgcospt = FORcosp 22h ogg /E 3 SOV There is thus a displacement to the east of 22s v=. 3% 8 64x 500 x 1x 98 26 cm. 8g 66 13 1.118 1119 1.120 1.121 Laws of Conservation of Energy, Momentum and Angular Momentum. As F is constant so the sought work done A= Fart F-@-7) > oo > le > eee or, A= (3i+4j )-[(2i-3j )-(@+2j )]= Gi+4j )-(i-5j )=17F Differentating v (s) with respect to time vs fa ds. ea @ nt ve Ve 2a (As locomotive is in unidrectional motion) 2 Hence force acting on the locomotive F =m w= 2 Let, at v= Oat ¢ = 0 then the — covered during the first ¢ seconds 1? a_i @ 2 2S 2. zz 4 i (a 4?) mat? Hence the sought work, A = Fs= —— yr ‘We have 2 To dma as? of, Fm 22 w Differentating Eq. (1) with respect to time 4as 2as 2vm,= Ayo w= Q Hence net acceleration of the particle = we Vidaw = ) +(222) - 28 Vie oaRye m mR m Hence the sought force, F = mw = 2asVi+ (s/R)° Let Fi makes an angle @ with the horizontal at any instant of time (Fig.). Newton’s second law in projection form along the direction of the force, gives : F = kg cos 6 + mg sin 0 (because there is no acceleration of the body.) eee = As Ft d7"the differential work done by the force F’, dA= F-d7™ Fds, (where ds = | d7"|) N = kong ds (cos @) + mg ds sin 8 = kmg dx + mg dy. 1 h Hence, A = long desme fy 0 0 a = kng1+mgh= mg (kl +h). 67 1122 Let s be the distance covered by the disc along the incline, from the Eq. of increment of ME. of the disc in the field of gravity : AT+ AU = Aj, ry 0+ (- mgs sin a) = — kang cos as - king I kl - 5" Sna-keosa a Hence the sought work Ag, = ~ kg [s cos 0 +1] _ king ‘ Ag ~7 get [Using the Eqn. (1)] On puting the values A, = -0.05 J 1123 Let x be the compression in the spring when the bar m, is about to shift. Therefore at this moment spring force on m, is equal to the limiting friction between the bar m, and horizontal floor. Hence xx= km,g [where « is the spring constant (say)] (1) For the block m, from work-energy theorem : A= AT = 0 for minimum force. (A here indudes the work done in stretching the spring.) so, Fr-hn2—kngx= 0 or k5a F-kmg Q, From (1) and (2), m ro tele’) 1.124 From the initial condition of the problem the limiting fricition between the chain lying on the horizontal table equals the weight of the over hanging part of the chain, i.e. Anlg= kA(1-%) lg (where 4d is the linear N mass density of the chain) — So, ea q@ Fs Let (at an arbitrary moment of time) the length ‘of the chain on the table is x. So the net friction force between the chain and the table, at this Ax moment : 3 Adx) f,= AN= kixg 2) 4 The differential work done by the friction forces : dha Fd ~f,ds~ -kaxg-d)= de( 7h \eae ® (Note that here we have written ds = -dr., because ds is essentially a positive term and as the length of the chain decreases with time, dx is negative) Hence, the sought work done o - a a en; pe A Sf re poee (=n) ne -133 Q-n)! 68 L125 1.126 1.127 The velocity of the body, ¢ seconds after the begining of the motion becomes v= ¥> +g. The power developed by the gravity (m g’) at that moment, is P= mg" v= m(g”- v9 + 8°t) = mg (gt- vy sina) qa) As mg” is a constant force, so the average power A_ mg? ar”

= oe —— tT where Ar” is the net displacement of the body during time of flight. As, mg"L Ar” so

= 0 ‘We have ee at?, ot, v= VaRt, t is defined to start from the begining of motion from rest. So, 0,2 Ba VaR Instantaneous power, PaF y= m (Ww, le, + Wy iy, )- (VaR th, ), (where ii, and i, are unit vectors along the direction of tangent (velocity) and normal respectively) So, P= mw,VaR t= ma Rt Hence the sought average power ' ' f Pdt f ma Rt dt

e SB ' t fe 0 _maRt? | maRt 2t 2 Let the body m acquire the horizontal velocity vy along positive x — axis at the point O. (a) Velocity of the body t seconds after the begining of the motion, ve twee (vy—ket)im q@ Instantaneous power P= Fv (- longi): (vy kgt) i= — kang (vo - ket ) From Eq. (1), the time of motion t= v9/kg Hence sought average power during the time of motion 1 Hence kang vo Jf He 75 — bee ya ee : F 7 ~2W (On substitution) From F,= mw, — kg = mw, = ee or, v,dv,= -kgdx = - agxdx 1128 1.129 1130 To find v (x), let us integrate the above equation v x fu - ag f rae or, P= y-age @ % 0 Now, P- Fv -maxgVve-agx* Q2) For maximum power, fee ) = 0 which yields x= Putting this value of x, in Eq. (2) we get, Prag - im vivag Centrifugal force of inertia is directed outward along radial line, thus the sought work n Yo V2ag Ae J mor dr= § mat G - rn) = 0:20T (On substitution) 4 Since the springs are connected in series, the combination may be treated as a single spring of spring constant. KK Ky +k, From the equation of increment of ME., AT+AU=A., K O+twalt= A, o, A=t *% \ap 2 2 Ky +} First, let us find the total height of ascent. At the beginning and the end of the path of velocity of the body is equal to zero, and therefore the increment of the kinetic energy of the body is also equal to zero. On the other hand, in according with work-energy theorem AT is equal to the algebraic sum of the works A performed by all the forces, i.e. by the force F and gravity, over this path. However, since AT= 0 then A = 0. Taking into account that the upward direction is assumed to coincide with the positive direction of the y-axis, we can write h kh asf ona * Se-mow s - me fa -2ay) dy= mgh(1-ah)= 0. 0 whence h= 1/a. The work performed by the force F over the first half of the ascent is nr nr Ars f Fay amg fay) ay 3 mg/4a. 0 0 The corresponding increment of the potential energy is AU= mgh/2= mg/2a. 70 1.131 1,132 From the equation F, = 2 we get F,= [-3+3] (a) we have at r= ro, the particle is in equilibrium position. ie. F,= 0 80, ro= a To check, whether the position is steady (the position of stable equilibrium), we have to satisfy @u ale @U_ [6a 2b We have a (e-3| Putting the value of r= r= 22, we get daUe (bs ra ‘se’ (as a and b are positive constant) So cule : dr? 8a’ which indicates that the potential energy of the system is minimum, hence this position is steady. : aq _[ 2b (b) We have F--2. 33] For F, to b i #, 0 ‘or F, to be maximum, = "= 3a -3 So, r= PF and then Fig)” 57> As F, is negative, the force is attractive. (a) We have au -aU F,- ~~ -2a0x and B= = -2By So, Fe 2axi-2Byiand, Fe 2Voexrepy () For a central force, 7 F= 0 aa >of Here, rxF= ity] )x(-2oxi-2By)) = -2Bxyk-2axy(k)«0 Hence the force is not a central force. ()) As U= ox? + py? au -au So, Fo Gyn ax and Fy= SO = -2By. So, FeVEsF =V4er+4py According to the problem F=2Va7x'+ py’ = C (constant) 4133 1144 1.135 71 or, esp y= ¢ a. | or, ea" axe” Fey) Q) Therefore the surfaces for which F is constant is an ellipse. For an equipotential surface U is constant. So, ax’+By* = Cy (constant) mye or, VE + Jt - an = Ky (constant) Hence the equipotential surface is also an ellipse. Let us calculate the work performed by the forces of each field over the path from a certain point 1 (x,, y,) to another certain point 2 (x,, y2) 5 @) dA= F- dF ayivd?™ ayde or, A= a fra > =. > a (i) dA= F-d7™ (axi+byi )-d7™= axdx + bydy 4 % Hence An f axdx+| bydy tee nm In the first case, the integral depends on the function of type y (x), ie. on the shape of the path. Consequently, the first field of force is not potential. In the second case, both the integrals do not depend on the shape of the path. They are defined only by the coordinate of the initial and final points of the path, therefore the second field of force is potential. Let s be the sought distance, then from the equation of increment of M.E. AT +AU = Ay, (0-58) + messin - kang cosas % or, s= 5p / (ina + keos a) ~ ke ae 4 2(e+ tana) yp =~ kang cos as = Velocity of the body at height h, v, = V2g (HA), horizontally (from the figure given in the problem). Time taken in falling through the distance h. t= V 2 {as initial vertical component of the velocity is zero.) Now s=v,t= Vig@ith V2 - Vat=F 1.136 1.137 For Spay 4 (Hh - 1?) =0, which yields h= # Putting this value of 4 in the expression obtained for s, we get, Swoax = H To complete a smooth vertical track of radius R, the minimum height at which a particle starts, must be equal to Sr (one can proved it from energy conservation). Thus in our problem body could not reach the upper most point of the vertical track of radius R/2. Let the particle A leave the track at some point O with speed v (Fig.). Now from energy conservation for the body A in the field of gravity : i 7 me[A-3C+sin)]- dm? cere (ty From Newton’s second law for the particle at the point O; F,= mw,, mn Ww But, at the point O the normal reaction N = 0 N+mgsin0= So, ve Bh sin 8 (2) From (3) and (4), sin 0= 2 and v= Y ea After leaving the track at O, the particle A comes in air and further goes up and at maximum height of it’s trajectory in air, it’s velocity (say v’) becomes horizontal (Fig.). Hence, the sought velocity of A at this point. non 2/8 v' = vcos (90 - 6) = vsinO= 3 3 Let, the point of suspension be shifted with velocity v, in the horizontal direction towards left then in the rest frame of point of suspension the ball starts with same velocity horizontally towards right, Let us work in this, frame. From Newton’s second law in projection form towards the point of suspension at the upper most point (say B) : 2 v my; m mg+T= 7" of, T= 7 -mg @) Condition required, to complete the vertical circle is that T= 0. But (2) im} = mg (2) +3 mii So, ve= yi-4gl @) 1.138 1.139 73 From (1), (2) and (3) v2 — 4 gl T= MUA Ae) geo or, v,= V5 gl Thus YA (wisy = V5 gt From the equation F, = mw, at point C oo 7 (4) s Again from energy conservation imi Sw + mg 6) From (4) and (5) T= 3mg Since the tension is always perpendicular to the velocity vector, the work done by the tension force will be zero. Hence, according to the work energy theorem, the kinetic energy or velocity of the disc will remain constant during it’s motion. Hence, the sought time t=, where s is the total distance traversed by the small disc during it’s motion. 0 Now, at an arbitary position (Fig.) ds = (Iy-R0)d®, UR s0, s=f G-R 840 0 2 2 o: 8 See te i 2K? 2R i Hence, the required time, f= IRV It should be clearly understood that the only uncompensated force acting on the disc A in this case is the tension T, of the thread. It is easy to see that there is no point here, relative to which the moment of force T is invarible in the process of motion. Hence conservation of angular momentum is not applicable here. Suppose that Al is the elongation of the rubbler cord. Then from energy conservation, AU,, + AUy= 0 (as AT = 0) = mg (1+ A) + 2x Al? = 0 or, 5x Al? — mg Al~ mg! =0 74 1.140 1.141 mgt Vf (mg) +4xSmgl z r] oe ke Zed or, al = xS= 147 1278 2 Since the value of V1 + a is certainly greater than 1, hence negative sign is avoided. — mel, ./ za | So, ue (1, 14 ng When the thread FA is burnt, obviously the speed of the bars will be equal at any instant of time until it breaks off. Let v be the speed of each block and 8 be the angle, which the elongated spring makes with the vertical at the moment, when the bar A breaks off the plane. At this stage the elongation in the spring. Al= hy sec @ ~ hy = ly (sec 8 ~ 1) Q) Since the problem is concerned with position and there are no forces other than conservative forces, the mechanical energy of the system (both bars + spring) in the field of gravity is conserved, ic. AT+ AU = 0 So, 2 G*) +413 (6ec0-1)?— mgiptan 0 = 0 Q From Newton’s second law in projection form N along vertical direction : 8 mg = N +x Iy (sec @ - 1) cos @ Klo Gxo-t But, at the moment of break off, N = 0. r Hence, x J, (sec 8-1) cos 8 = mg K1y)- mg or, cos 0 = ce @) "™, a 78, simultaneous solution\of (2) and (3) yields : 3 fo ve = 17m/s. Obviously the elongation in the cord, Al= /, (sec 6 - 1), at the moment the sliding first Starts and at the moment horizontal projection of spring force equals the limiting friction. So, x, Alsin 8 = KN qa (where x, is the elastic constant). KAL From Newton’s law in projection form along vertical direction : k, Al cos 0 +N = mg. or, N= mg~x, Alcos® From (1) and (2), (2) x, Alsin @ = k(mg - «x, Al cos @) mg 1.142 1.143 75 or, oc gaa *1™ Aisin @ + k Al cos 0 From the equation of the increment of mechanical energy : AU + AT = Ay, 1 or, (:na")- 4, king Al? 2. Al (sin 8 + & cos 0) dang ly (sec 8 - 1) 2 (sin © — k cos ®) Let the deformation in the spring be Al, when the rod AB has attained the angular velocity «. From the second law of motion in projection form F,= mw, . or, = Ay, Thus Ay, = = 0-093 (on substitution) 2 mor I, KAl= mo? (Ip+Al or, Al= (y+ AD Par 2 From the energy equation, A, = jm + 7 x Al? 13 aol = yma? (Ip + Al? +5 x Al We know that acceleration of centre of mass of the system is given by the expression. — > Mm Witmw, Wom Tay +m, Since Wim - <> __ (my -m,) W Wem @ my +m, Now from Newton’s second law F'= mis, for the bodies m, and m, respectively. T+ m, a= m,¥%, @ and T+ m,g% mwy=— mi, (3) Solving (2) and (3) wn mim : my +m, 4 76 1.144 1.145 1.146 Thus from (1), (2) and (4), a> _ (m-my 3 (m, + my As the closed system consisting two particles m, and of m, is initially at rest the C.M. of the system will remain at rest. Further as m= m,/2, the C.M. of the system divides the line joining m, and mz at all the moments of time in the ratio 1 : 2. In addition to it the total linear momentum of the system at all the times is zero. So, p; = — Dp, and therefore the velocities of m, and m, are also directed in opposite sense. Bearing in mind all these thing, the sought trajectory is as shown in the figure. First of all, it is clear that the chain does not move in the vertical direction during the uniform rotation. This means that the vertical component of the tension T balances gravity. As for the horizontal component of the tension T, it is constant in magnitude and permanently directed toward the rotation axis. It follows from. this that the C.M. of the chain, the point C, travels along horizontal circle of radius p (say). Therefore we have, TcosO= mg and Tsin0 = mo” p Thus p= £228. ag cm w and T= 782 SN cos 6 (a) Let us draw free body diagram and write Newton’s second law in terms of projection along vertical aud horizontal direction respectively. Neos a -mg + frsina= 0 qa froosa-Nsina = mw?! (2) From (1) and (2) rcos = 2S (fr sin a+ mg) = mo?! cos Ot 8 1.147 2 So, fr= me sna ®cosa)= oN @) (b) For rolling, without sliding, frs kN but, N= mg cos a-mw7Isina o71 2 mg | sina +S 00s at | = k (mg cos o- mc? sin ct) [Using (3)] Rearranging, we get, mo? l(cosa+ksina) = (kmg cos a - mg sina) Thus wsVg(k-tana)/(1+ktana)! = 2 rad/s (a) Total kinetic energy in frame K’ is 1 —- > 1 oe T= 5m, (H-V P+ 5m (-V) This is minimum with respect to variation in V, when so 0, ie. m, (4 -VY +m, (1, -V)=0 ytm,V, > my + ve Mut" or i m,+m, Hence, it is the frame of C.M. in which kinetic energy of a system is minimum. (b) Linear momentum of the particle 1 in the K’ or C frame = oy I oe Ble m (~~ AM aw) ee my my or, Pi= #(¥,~-¥2), where, p= +m, reduced mass Similarly, P= w(-%) So, [Pil= [Pol= B= wy where, v,¢= 1¥7-¥%I (3) Now the total kinetic energy of the system in the C frame is eee ee a 2 T-T+h- £-E- a fol Hence Te guvme 5H | A-a| 78 1.148 1.149 1.150 To find the relationship between the values of the mechanical energy of a system in the K and C reference frames, let us begin with the kinetic energy T of the system. The velocity of the i-th particle in the K frame may be represented as ¥;= ¥; + Vo. Now we can write T= Sy Emvt= D $m (F400): O47) ta = yl 2 =D ymBsvey mite yD ome Since in the C frame J) m;¥;= 0, the previous expression takes the form Fri mye Fed mv? (Gince according to the problem v¢= V) Q) Since the internal potential energy U of a system depends only on its configuration, the magnitude U is the same in all refrence frames. Adding U to the left and right hand sides of Eq. (1), we obtain the sought relationship E=E+ im v? As initially U= 0 = 0, so, E= T From the solution of 1.147 (b) Fs ult-H1, As WL z1mim Thus T- Im, +m 01t) Velocity of masses m, and m,, after f seconds are respectively. Wy = Weg and Hy = y+ et Hence the final momentum of the system, — = os ~ Pe mV, +m,Vz = Mm, V, +m, V2 + (mM, +m) gt = potmat, (where, po= mV, +m, ¥; and m= m,+m,) And radius vector, re were hi (my V+ my vg) t else (m, +m) ey | >, 1a» —» MV, +m, Vv. = Wore S et, where ee o 739 1.151 After releasing the bar 2 acquires the velocity v,, obtained by the energy, conservation : 1.152 1,153 1 1 m= aKX oF, y= XV = (a) 2 Thus the sought velocity of C.M. osmzVm Vig m+m,—~ (m+m,) Vom = Let us consider both blocks and spring as the physical system. The centre of mass of the system moves with acceleration a = as towards right. Let us work in the frame of 1 2 centre of mass. As this frame is a non-inertial frame (accelerated with respect to the ground) we have to apply a pseudo force m, a towards left on the block m, and m, a towards left on the block m, As the center of mass is at rest in this frame, the blocks move in opposite directions and come to instantaneous rest at some instant. The Mp a elongation of the spring will be maximum or minimum at this instant. Assume that the block lm Losmoon ‘m, is displaced by the distance x, and the block mz, through a distance x, from the initial positions. From the energy equation in the frame of C.M. AT+U=A,,, (where Aj, also includes the work done by the pseudo forces) Here, AT=0, U=ZkG, +2) and F-m,F m,F m, F (x, +x,) We) ee m, +m, m, +m, m,+m, 1 2m (1 +) F or, 5 ely +2) am . 2m,F 0, Hyty = 0 00 2 + 22 = Fos my 2m, F Hence the maximum separation between the blocks equals : /) + ————~ K(m, +m) Obviously the minimum sepation corresponds to zero elongation and is equal to Jy (a) The initial compression in the spring Al must be such that after burning of the thread, the upper cube rises to a height that produces a tension in the spring that is atleast equal to the weight of the lower cube. Actually, the spring will first go from its compressed 80 1.154 State to its natural length and then get elongated beyond this natural length. Let / be the maximum elongation produced under these circumstances. Then xl = mg () Now, from energy conservation, 1 2 di? Ze A?= mg (Ale D+ 5K! (2) (Because at maximum elongation of the spring, the speed of upper cube becomes zero) From (1) and (2), 2 ay? 2mga Smeg 0 oy b= 2, =H « Therefore, acceptable solution of Al equals oz (b) Let v the velocity of upper cube at the position (say, at C ) when the lower block breaks off the floor, then from energy conservation. iat 22) piv? = 5x (Al? = 1?) mg (1+ Al) (where = mg/x and Al = 78) 2 or, ve 3278 Q) « mv+0 At the position C, the velocity of C.M; vo= "> . 5 ake, the C.M. of the system (spring+ two cubes) further rises up to A yoy Now, from energy conservation, 5m) v= Om) g bres But, uptil position C, the C.M. of the system i has already elevated by, (Al+1m+0 4m Oe eee = Hence, the net displacement of the CM. of the system, in upward direction 8 mg Ayc™ AYa+A¥a= 1 K Due to ejection of mass from a moving system (which moves due to inertia) in a direction perpendicular to it, the velocity of moving system does not change. The momentum change being adjusted by the forces on the rails. Hence in our problem velocities of buggies change only due to the entrance of the man coming from the other buggy. From the 1.155 1.156 1.157 81 Solving (1) and (2), we get mv My Me uta and v, = M-m As Mth vand tty” => -mv = My So, 1" Grom 4 2" Geom From momentum conservation, for the system “rear buggy with man” (M+m)ip= mut ig) + Mig (@) From momentum conservation, for the system (front buggy + man coming from rear buggy) Mip+m (Ut vp) = (M+m) vp Mv = 0 m So, “e" M+m*M+m Putting the value of Vz from (1), we get @+ vg) et Or (M+ my (i) Let v7 be the velocity of the buggy after both man jump off simultaneously. For the closed system (two men + buggy), from the conservation of linear momentum, Mij+2m(i+i7)= 0 os i ip ® (ii) Let 0” be the velocity of buggy with man, when one man jump off the buggy. For the closed system (buggy with one man + other man) from the conservation of linear momentum : O= (M+m)v" +m(uv") (2) Let v3 be the sought velocity of the buggy when the second man jump off the buggy; then from conservation of linear momentum of the system (buggy + one man) : (M+m)v" = Miz +mU+i) @Q) Solving equations (2) and (3) we get or, (2M + 3m) a” (4) “2” (M+ m) (M+ 2m) From (1) and (4) %2 m 4 1 *2@tem >! Hence v,>v, The descending part of the chain is in free fall, it has speed v= V2 gh at the instant, all its points have descended a distance y. The length of the chain which lands on the floor during the differential time interval dt following this instant is vdt. 1.158 1.159 For the incoming chain clement on the floor : From dp, = F,dt (where y-axis is directed down) ' 4 0- (vit) v= F, dt or Fy= -hv= -2dgy Hence, the force exerted on the falling chain equals 2.v* and is directed upward. Therefore from third law the force exerted by the falling { chain on the table at the same instant of ae = pee time becomes 2 v? and is directed downward. v Since a length of chain of weight (yg) already lies on the table the total force on the floor is (2Ayg) + (yg) = (3Ayg) or the weight of a length 3y of chain. Velocity of the ball, with which it hits the slab, v= V2 gh After first impact, v’ = ev (upward) but according to the problem v= WP so e= a q@) and momentum, imparted to the slab, = mv-(- mv’) = mv(1 +e) Similarly, velocity of the ball after second impact, via evm ey And momentum imparted = m(v'.+v’)= m(L+e)ev Again, momentum imparted during third impact, = m(1+e)e*v, and so on, Hence, net momentum, imparted = mv (1 + e) + mve (1 +) + mve” (1 +e) +... = mv(l+e)(ltete+...) (1+e) = ee) , (from summation of GP) (1 += n = v2 gh : = m2 gh / (q+ 1)/(n- 1) (Using Eq. 1) (4) = 0-2kg m/s. (On substitution) (a) Since the resistance of water is negligibly small, the resultant of all external forces acting on the system “a man and a raft” is equal to zero. This means that the position of the C.M. of the given system does not change in the process of motion. ie. 7o= constant or, Are= 0 ie. Sm; Arr= 0 or, m (Ane + Ay, +M bie = 0 Thus, m(T" +1)+Mie 0, or, T=- al” : = m+M (b) As net external force on “man-raft” system is equal to zero, therefore the momentum of this system does not change, So, O= m[” (+H O)]+MA) 1159 1.160 1161 1.162 83 (a) Since the resistance of water is negligibly small, the resultant of all external forces acting on the system “a man and a raft” is equal to aero. This means that the position of the C.M. of the given system does not change in the process of motion. ie. 7¢= constant or, Arc= 0 ie. Sy m,Ar= 0 or, a ed + Aig) + M ry, =0 | ie Thus, m(I"'+1)+MI1= 0, or, 10-7 (b) As net external force on “man-raft” system is equal to zero, therefore the momentum of this system does not change, So, O= m[V" (H+ HO ]+M() or, wy(@)= - mila qa) As ¥” (0) or ¥5(¥) is along horizontal direction, thus the sought force on the raft Madi Mm dv") dt m+M_ dt Note : we may get the result of part (a), if we integrate Eq. (1) over the time of motion of man or raft. In the refrence frame fixed to the pulley axis the location of C.M. of the given system is described by the radius vector ar M Arig + (M =m) Arig my +m Ary ‘= 2M > a But Ary = ~ Ara my and ATE = AFoagm + ATor-m (4-mn) +m mI” Thus Arca ou Note : one may also solve this problem using momentum conservation. Velocity of cannon as well as that of shell equals V2 gi sina down the inclined plane taken as the positive x - axis. From the linear impulse momentum theorem in projection form along x - axis for the system (connon + shell) ic. Ap, = F, At: pcosa-MV2gisina = Mgsina At (as mass of the shell is neligible) poosa-MV 2glsina or, At= = Mgsino. From conservation of momentum, for the system (bullet + body) along the initial direction of bullet mv m+M mvy= (m+M)v, of, v= 84 1.163 1.164 When the disc breaks off the body M, its velocity towards right (along x-axis) equals the velocity of the body M, and let the disc’s velocity’in upward direction (along y-axis) at that moment be v', From conservation of momentum, along x-axis for the system (disc + body) mv my= (m+M)v, ot vgn Se (@) And from energy conservation, for the same system in the field of gravity : re ae ; zm = s(m+M) vz +> mvs + meh , where A’ is the ae of break off point from initial level. So, Lin mv 1 2» , ‘ zm = Some) - an using (1) y2 , or, vee PH ime 7) -2gh' Also, if h” is the height of the disc, from the break-off point, then, vom 2 gh" 2 mv So, 2g (h" +h')= ve oA Hence, the total height, raised from the initial level Me oN th J Mem) (a) When the disc slides and comes to a plank, it has a velocity equal to v= V2 gh. Due to friction between the disc and the plank the disc slows down and after some time the disc moves in one piece with the plank with velocity v’ (say). From the momentum conservation for the system (disc + plank) along horizontal towards right : my m+M Now from the equation of the increment of total mechanical energy of a system : mv= (m+M)v' or v' 1 a_i (M+ m)v? mv = Ay, my? 1 1 or, =(M+m -imv= Ay, x6 Gna Me z " so, Hence, 1.165 1.166 85 (b) We look at the problem from a frame in which the hill is moving (together with the disc on it) to the right with speed u. Then in this frame the speed of the disc when it just gets onto the plank is, by the law of addition of velocities, ¥ = u +V2gh. Similarly the common speed of the plank and the disc when they move together is = m V = us Vig vi 1 ze Then as above A, = yim+M vom a - Mu’ i 2 ums 1 21 = Z(m+M) ferme MU V2Bh + anage 2h} ~ 3 MO 5 m 20 VT ~ mg We see that A, is independent of u and is in fact just - 4 gh as in (a). Thus the result obtained does not depend on the choice of reference frame. Do note however that it will be in correct to apply “conservation of enegy” formula in the frame in which the hill is moving. The energy carried by the hill is not negligible in this frame. See also the next problem. In a frame moving relative to the earth, one has to include the kinetic energy of the earth as well as carth’s acceleration to be able to apply conservation of energy to the problem. In a reference frame falling to the earth with velocity v,, the stone is initially going up with velocity v, and so is the earth. The final velocity of the stone is 0 = v,—gt and that of the earth is v, + Fret (M is the mass of the earth), from Newton's third law, where t = time of fall. From conservation of energy dmg + Sag + mgh = 7M (r0 + i) 1 mi Hence 3% (» + ii) = mgh Negecting 7 in comparison with 1, we get vp = 2gh or v, = V2gh The point is this in earth’s rest frame the effect of earth’s accleration is of order 7 and can be neglected but in a frame moving with respect to the earth the effect of earth’s acceleration must be kept because it is of order one (ie. lange). From conservation of momentum, for the closed system “both colliding particles” myvz + mv, = (m, +m)" ge Mut mah 187227} +24j- 68). 797 ze 3 or, v= my +m, Hence [T= V144+16 m/s= 46m/s 1.167 1.168 For perfectly inelastic collision, in the C.M. frame, final kinetic energy of the colliding system (both spheres) becomes zero. Hence initial kinetic energy of the system in C.M. frame completely tums into the internal energy (Q) of the formed body. Hence ze |a- ay 2 Now from energy conservation AT™= -Q = -hu r- In lab frame the same result is obtained as LL (aie my 1 A 2 sy2 arn 5 mee 5m Lael? +m 15 i =- ely - 4 (a) Let the initial and final velocities of m, and m, are ui, u, and V,, V2 respectively. Then from conservation of momentum along horizontal and vertical directions, we get : mu, = m,v, cos @ 0) and my, = my, sin® Q tv Squaring (1) and (2) and then adding them, ub miy3 = my (ui + vt) ~O> wan nee eee yee. Now, from kinetic energy conservation, 9 \. 1 1 1 . Lin ade Lm, vba dim v2 ®) — mi or, m(-v2)= my= m, a +v2) [Using (3)] 2 m: mM: or, @i1-—]= Bf14+— m, m, Y; . m, - (2) ee @ hed My + So, fraction of kinetic energy lost by the particle 1, tt gman am ve 2 — 21-4 zm ua wy mm, - Mm; 2m, : ~ 1-24 = [sing (4)] 6) m,+m, my +m, (b) When the collision occurs head on, mu, = my, + mV, () and from conservation of kinetic energy, 1.169 87 or, (6) or, uy (f+ m7) Fraction of kinetic energy, lost 2 mn _ 4mm, =1- =" 1- sing (6) Z (Bam ae [Using (6)] (a) When the particles fly apart in opposite direction with equal velocities (say v), then from conservatin of momentum, m,u+0= (m,-m)v () and from conservation of kinetic energy, Lads ban Pod? or, mw = (m,+m,) 7 Q) From £q. (1) and (2), Pu? m mw = (m+ U7, aoe Co or, m,-3m,m,= 0 Hence “tat asm, 00 oo es (>) When they fly apart symmetrically relative to the initial motion direction with the angle of divergence @= 60°, From conservation of momentum, along horizontal and vertical direction, im, uy = m, v, 608 (8/2) + ma v, 008 (0/2) @ and im, v, sin (0/2) = mv, sin (0/2) or, mV, = m2 Vv, (2) Now, from conservation of kinetic energy, 1 az 1 FMM +O= om yt amy @) From (1) and (2), mv amy ty = 05 (8/2) [my » + ma} = 2m, v, cos (8/2) 1.170 So, u, = 2-v, cos (0/2) (4) From (2), (3), and (4) 2% 4m, cos? (8/2) v3 = m, v2 4 727 2 a or, 4cos?(0/2)= 1+— m mM, or, te cos? 2-1 m, 2 m, and putting the value of 8, we get, a 2 If (¥4,,%y) are the instantaneous velocity components of the incident ball and (ve, Vay) are the velocity components of the struck ball at the same moment, then since there are no external impulsive forces (i.e. other than the mutual interaction of the balls) ‘We have Usina= vy , vy=0 MU COSA. = MV, + M Voy The impulsive force of mutual interaction satisfies d F d gwds a> ay ad) (F is along the x axis as the balls are smooth. Thus Y component of momentum is not transferred.) Since loss of KLE. is stored as deformation energy D, we have 1. 1g Ln D zm pm 5mv, 1 = me cos’ — smvyy ~ sv, : x miaPeosta- mv? — (mucosa mv,,)? | -= [ 2mucosar,,~ 2m'v2 ] = m(vigoose~ v,.") 2 _ Weos’a _ ( ucosa : 4 Pe ‘We see that D is maximum when u Sn Ma Zoos mu’cos’a. and Des 4 4 > Then = = deosta = b u 1p 2 4 ym On substiuting a= 45° x 1.171 From the conservation of linear momentum of the shell just before and after its fragmentation 1.172 Were ee @) where ¥;, ¥5 and ¥3 are the velocities of its fragments. From the energy conservation 3nv?=v3 +3403 Q Now Vor Vie = Vi Ve =v @ where 72= "= velocity of the CM. of the fragments the velocity of the shell. Obviously in the C.M. frame the linear momentum of a system is equal to zero, so oe Seo Vy+v,+¥3=0 4) Using (3) and (4) in (2), we get By = VET) + OEP FOL V- VE)? = v4 272 4 292 4 207 or, 252 + 25, F, cosO + 273+3(1-n) =0 () If we have had used ¥)=-¥; ~ Vy, then Eq. 5 were contain 4, instead of ¥, and so on. The problem being symmetrical we can look for the maximum of any one. Obviously it will be the same for each. For 9,10 be real in Eq. (5) 453 cos’ = 8(273 + 3 (1 - 1) v7) or 6(n - 1) & (4 - cos”0)F3 So, Bev YEO? ot Fay V2-0 v Hence 2 (au) |V%* Vala, =¥+V2 (0-1) vev(teV2q@-0 =1km/s Thus owing to the symmetry Y yguax) ¥2 as)" Yanan) 7 ¥ (1 +¥2(n = 1)) = 1 km/s Since, the collision is head on, the particle 1 will continue moving along the same line as before the collision, but there will be a change in the magnitude of it’s velocity vector. Let it starts moving with velocity v, and particle 2 with v, after collision, then from the conservation of momentum mu = mv, + mv, Of, Us Vv, +V2 () And from the condition, given, 1 2 av? +} mv zyme dee vie qe 2+. 1-+2 Smut uw or, vievi= (L-n) i? Q) From (1) and (2), vi+ uv? = -n) or, Vaew-2uv, += (L-n)u? 90 1.173 1.174 or, wi-wutnu= 0 aa So, ve Que eS = 3 [e2ViP-an? l- dua *vI-%m) Positive sign gives the velocity of the 2nd particle which lies ahead. The negative sign is correct for vy. So, ¥, = ju (1-VI=2y ) = 5 m/s will continue moving in the same direction. Note that v,= 0 if n= 0 as it must. Since, no external impulsive force is effective on the system “M + m’”, its total momentum along any direction will remain conserved. So from p, = const. u mu = Mv, cos ®@ or, “ed Q) and from p, = const v= My,sinO ot, v= My, sinO= wtan0, [using (1)] Final kinetic energy of the system 7,= Lmisiod And initial kinetic energy of the system= zoe T, -T; % change = Te 100 So, 2 Mee dm tan? 0 += sue, =F mu M? cos*@ 2 . x 100 7 2 2” u ae 2 im ao 2 tan?044 at see 0 ju i “a 13 2" - S . oa +7 sec" 0 — 1) x 100 and putting the values of @ and a , we get % of change in kinetic energy= - 40 % (a) Let the particles m, and m, move with velocities vy and V3 respectively. On the basis P 1 2 1 Tesp y. of solution of problem 1.147 (b) Bourg oo | H-¥| 1175 1.176 91 As WLI, m So, Bo uVied where y= L™ mm, +m, (©) Again from 1.147 (b) ae Lye xo 2 P= Swe Sut] So, F-Sucieyd From conservation of momentum P=P +P > met? , Ui ' so (-Pr') = Pi-20, py’ e090, + py? = py? From conservation of energy a : py? . Py? Im, ~ Im, * Im, Eliminating p,' we get ry mz ' q my O= py (+ = ~ 2p;' p,cos0, + py (\- | This quadratic equation for p,' has a real solution in terms of p, and cos 0, only if 2 mi tenn -F my i SOY. 2 mi or sin? 0, <> my oo 8 7 . my ‘ my or sin®, v'? + v’ so, 0",-¥°)>0 and therefore angle of divergence < 90°. Suppose that at time 4 the rocket has the mass m and the velocity v”, relative to the reference frame, employed. Now consider the inertial frame moving with the velocity that the rocket has at the given moment. In this reference frame, the momentum increament that the rocket & ejected gas system acquires during time dt is, dp= mais udti= F dt on mite F- wi 1.179 1.180 1.181 93 According to the question, F= Oandp = — dm/dt so the equation for this system becomes, mit, ini» dt” dt As dvt}i” so, mdv= -udm. Integrating within the limits : : s 1fa-- de Ye in ™ u mn 0 mo Thus, v= uin— m mo As dv't{ i so in vector form v= -#In > According to the question, F (external force) = 0 So, nt. he , a a" As at iy so, in scalar form, mdv= -udm wdt dm or, “Ss u m Integrating within the limits for m (1) wt dn v ee ee u m u my my Hence, m= me“) As F= 0, from the equation of dynamics of a body with variable mass; av” adm wi o ma eG dv= iv Now dv"} jii’and since i"L, ¥; we must have | dv"| = a (because vp is constant) where do. is the angle by which the spaceship turns in time df. udm So, -u Meyda o daw -4+™ m Vo m . _ fens (3) % 94 1.182 1.183 1.184 We have —-= -p or, dm= - pdt dt : Integrating fame -u fa of, m= my-pt ™, 0 As u’= 0 so, from the equation of variable mass system : dv > dvi > (mp - vt) G = F or, a7 0s F/m- ws) - : or, famF a. (2g - wt) 0 Hence oe F inf —"o_ Bo [my pe Let the car be moving in a reference frame to which the hopper is fixed and at any instant of time, let its mass be m and velocity v- Then from the general equation, for variable mass system. dv" => —sdm mat Ft dt We write the equation, for our system as, dv" > dm —- = ma FOS as, we 70 qd) So 4 iy =F BR and v= = on integration. But m= m+ pt = > Fr 50, ve m (: *m mo > Thus the sought acceleration, w= 2. SS It milton ) 0 Let the length of the chain inside the smooth horizontal tube at an arbitrary instant is x. From the equation, —» = —»dm mw= F+u dt 1.185 95 as T= 0, F tt @; for the chain inside the tube Axw= T where Met (a) Similarly for the overhanging part, AW Tr w= 0 ooo Thus mw = F T or Ahw= Khg-T (2) h From (1) and (2), hg. dv AQ+A)w= Ahg or, +h)v Tem hg B , dv or, (+iv Capt Bh [As the length of the chain inside the ube decreases with time, ds = - dx.] de or, vdv= -gh5 » ° Integrating, J vdv= -gh f ay 0 (lek) or, ee gh In (i) or v= VY 2h In (i) Force moment relative to point O ; N- an. 2b — Let the angle between M and N, a= 45° att = tye MeN @+b2)- 2b) IMIIN| Vat +0? tf 2bt0 2 43 2 2b" ty bty Vas bt 21, Var+b ty Then Jy So, 2b 4b= a? +bPtg oF, tom Vv £ (as fg cannot be negative) It is also obvious from the figure that the angle a is equal to 45° at the moment fo, when a= bi2, ie. fy = Va7b and N= 25 a 96 1.186 1,187 1.188 M(Q)=7kpe= (renga ‘xm 4 +3) = ma? sin (3+ ack y+ E-mu gt * sin (5 +a)@) 4 ivy gt? cosa (-E): 2 mvy 8 t? cos a Thus M (1) = 20 Thus angular momentum at maximum height : x vosina ie at t= t=———, 2 8g a a) (mi : m5) Fe [ sw acosa 37kg-m'/s Q Alternate : 777 Gee MO)=0 so, Mi= f Nac= f (Femz) o oO 2 <> Looe is J [eee ae maa (*o* me) 7 (a) The disc experiences gravity, the force of reaction of the horizontal surface, and the force R of reaction of the wall at the moment of the impact against it. The first two forces counter-balance each other, leaving only the g force R. It’s moment relative to any point of A the line along which the vector R acts or along normal to the wall is equal to zero and therefore oL > the angular momentum of the disc relative to = any of these points does not change in the given process. (b) During the course of collision with wall the position of disc is same and is equal to FZ, Obviously the increment in linear momentum of the ball Ap™= 2mv cos a.n Here, AM = 72, x Ap™= 2mv cos cn and directed normally emerging from the plane of figure Thus |AM|= 2mvl cosa (a) The ball is under the influence of forces T’ and m "at all the moments of time, while o-.. . - 9 3. moving along a horizontal circle. Obviously the vertical component of T balance m g’and 1.189 1.190 97 So the net moment of these two about any point becoems zero. The horizontal component of T, which provides the centripetal acceleration to ball is already directed toward the centre (C) of the horizontal circle, thus its moment about the point C equals zero at all the moments of time. Hence the net moment of the force acting on the ball about point C equals zero and that’s why the angular mommetum of the ball is conserved about the horizontal circle. (b) Let o be the angle which the thread forms with the vertical. Now from equation of particle dynamics : Tcosa= mg and Tsina = mw’ Isina 1 . or @ As |M | is constant in magnitude so from figure. |AM| = 2M cos a where M~ |M;|= |M,| = |xmv |= my I (as iio LV) Hence on solving cos 0. = Thus|AM| = 2 mv 1cos a= 2 mw I? sin a cos 2mgl /, e = a &] (using 1). , the reference point O moves in hoizontal direction During the free fall time t= t = (say towards right) by the distance V7. In the translating frame as M (0) =0, so o ~> AM= Mi,=F / iW) = (-Vuithj )xm[guj-Vi] o 2 = -mV girs mVh(+é) J (9) 2h) = . : -mve(F|Femvaed) = -mVhk Hence |AM|= mVA The Coriolis force is.(2m 0" x @). Here @ is along the z-axis (vertical). The moving disc is moving with velocity vy which is constant. The motion is along the x-axis say. Then the Coriolis force is along y-axis and has the magnitude 2m vo. At time ¢, the distance of the centre of moving disc from O is vot (along x-axis). Thus the torque N due to the coriolis force is N= 2mvg eve along the z-axis. 1.191 1.192 Hence equating this to a aM GT UMMOt or M= mvj ot? + constant. The constant is irrelevant and may be put equal to zero if the disc is originally set in motion from the point O. This discussion is approximate. The Coriolis force will cause the disc to swerve from straight line motion and thus cause deviation from the above formula which will be substantial for large t If = radial velocity of the particle then the total energy of the particle at any instant is i) Me mira thn E 1 2 oe @) where the second term is the kinetic energy of angular motion about the centre O. Then the extreme values of r are determined by 7 = 0 and solving the resulting quadratic equation pe KP Br + no we get From this we see that E=Kyi +7) Q) where r, is the minimum distance from O and r, is the maximum distance. Then dom} + Dkr} = k (A+) hi 2? fence, m= i) ‘Note : Eq, (1) can be derived from the standard expression for kinetic energy and angular momentum in plane poler coordinates : 1a 7 T=5mr +zmro M = angular momentum = mr” 6 The swinging sphere experiences two forces : The gravitational force and the tension of the thread. Now, it is clear from the condition, given in the problem, that the moment of these forces about the vertical axis, passing through the point of suspension N, = 0. Con- sequently, the angular momentum M, of the sphere relative to the given axis (z) is constant. Thus myg (I sin @) = mv 1 qa where m is the mass of the sphere and v is it s velocity in the position, when the thread x forms an angle 5 with the vertical. Mechanical energy is also conserved, as the sphere is 1193 1.194 1.195 99 under the influence if only one other force, i.e. tension, which does not perform any work, as it is always perpendicular to the velocity. So, imi +mglcos 0= im? (2) From (1) and (2), we get, Vg = V2gI7cos © Forces, acting on the mass m are shown in the figure. As N= mg’ the net torque of these two forces about any fixed point must be equal to zero. Tension T, acting on the mass m is a central force, which is always directed towards the centre O. Hence the moment of force T is also zero about the point O and therefore the angular momentum of the particle m is conserved about O. Let, the angular velocity of the particle be @, when the separation between hole and particle m is r, then from the conservation of momentum about the point O, : m (7p) ro = m(wr)r, or or a r Now, from the second law of motion for m, T= F= mor Hence the sought tension; mogrér moard Foe —_—= ne re F On the given system the weight of the body mis the only force whose moment is effective about the axis of pulley. Let us take the sense of @ of the pulley at an arbitrary instant as the positive sense of axis of rotation (z-axis) As M,(0)= 0, s0, AM, = M, (= f N,de ' So, M,@= f mgRdt= mg Rt Oo Let the point of contact of sphere at initial moment (t= 0) be at O. At an arbitrary moment, the forces acting on the sphere are shown in the figure. We have normal reaction N, = mg sin a. and both pass through same line and the force of static friction passes through the point O, thus the moment about point O becomes zero. Hence mg sin a: is the only force which has effective torque about point O, and is given by |N |= mgRsina normally emerging from the plane of figure. As M(t= 0)= 0, so, A= (= f Nar Hence, M(t)= Nt= mgRsinot 100 1.196 1.197 1.198 Let position vectors of the particles of the system be O and O’ respectively. Then we have, in +7 () nd 7; with respect to the points where 76 is the radius vector of O! with respect to O. Now, the angular momentum of the system relative to the point O can be written as follows; Mm ¥ (xt) - D (7 «Pi)+ > (exz) [using (1)] or, u- ia + (x P), where, BY 7 @ From (2), if the total linear momentum of the system, p’= 0, then its angular momen- tum does not depend on the choice of the point O. Note that in the C.M. frame, the system of particles, as a whole is at rest. On the basis of solution of problem 1.196, we have concluded that; “in the C.M. frame, the angular momentum of system of particles is independent of the choice of the point, relative to which it is determined” and in accordance with the problem, this is denoted by M. We denote the angular momentum of the system of particles, relative to the point O, by M,. Since the internal and proper angular momentum Mf, in the C.M. frame, does not depend on the choice of the point O’, this point may be taken coincident with the point O of the K-frame, at a given moment of time. Then at that moment, the radius vectors of all the particles, in both reference frames, are equal (7° = F;) and the velocities are related by the equation, Wn Viv, a where v7 is the velocity of C.M. frame, relative to the K-frame. Consequently, we may write, Me Sm (Hea e Dm (798) Fm (FH) o, Ms Mam (xi), as J) mrj= m72, where m=) m,. or, Mz f+ (rx mvp) = M+ (xP) From conservation of linear momentum along the direction of incident ball for the system consists with colliding ball and phhere myy= my! + 3 %y (1) where v’ and v, are the velocities of ball and sphere 1 respectively after collision. (Remember that the collision is head on). As the collision is perfectly clastic, from the definition of co-efficeint of restitution, vay 0-% 1= on, v= y= -% Q) 1.199 101 Solving (1) and (2), we get, 4v Vy Y= > directed towards right. @->-- my 4 i In the C.M. frame of spheres 1 and 2 (Fig.) Pi = -Py and |Dil= 173] = wly- HI Also, Fic = ~Tic, thus M = 2[7{-xDz] c Avy a As Fic L Diy so, Bea[} m2 a % (where 1 is the unit vector in the sense of 77x Pt ) mp mol Hence M= In the CM. frame of the system (both the discs + spring), the linear momentum of the discs are related by the relation, p;= —p5 at all the moments of time. where, By By= B= WM pe And the total kinetic ae of the system, T= swe [See solution of 1.147 (b)] Bearing in mind that at the moment of maximum deformation of the spring, the projection of Vy along the length of the spring becomes zero, i.e. Vj21(.)= 0 The conservation of mechanical energy of the considered system in the C.M. frame gives. 1 (m (3) 3 aera (al ra) (a) Now from the conservation of angular momentum of the system about the C.M., 1 (4) +X) m 3(2 G »)- 2a Jaro vol xy? x or, Yel 9) Ts - v(t “7 ~ volt-Z} as x<< ly Q) Using (2) in (1), amils-(-f) Jee? 2 or, ms] +) |-xe 0 mvox : or, i ~ «x, [neglecting x” / 12] mv% As xe 0, thus x= Kh 102 1.4 1.200 1.201 UNIVERSAL GRAVITATION We have Mi yMm tm a ee Thus ¥ v o=t=—"_ 2 7 rym/v? ym, (Here m, is the mass of the Sun.) Qmym, In x 667x107" x 197 x 10% So T= St = XOOe eee y (349 x 107) (The answer is incorrectly written in terms of the planetary mass M) = 194 x 10" sec = 225 days. For any planet MRot= — or w= yf R? So, T= =. 2nR?7/Vym, 372 T, Ry (a) Thus 7m (z) Ry 1s a So Be” Til Te) = (12)? = 5-24. ws ae ym, = {rv ) Viw TE, and Ry (“He ws . v2 (rm, )? 2x)? y, = (220% i T acer 7 where T= 12 years. m,= mass of ths Sun. Putting the values we get V, = 12:97 km/s 2 . vy (2erm, 2x pectin = Za (222%) (ie aa - (7) (ym, )'? = 245x104 km/s” 103 1.202 Semi-major axis= (r + R)/2 r+R 2 It is sufficient to consider the motion be along a circle of semi-major axis for T does not depend on eccentricity. r+R 2n\ 2 ae (again m, is the mass of the Sun) Hence T= y" = nV(r+R)/2ym, 1.203 We can think of the body as moving in a very elongated orbit of maximum distance R and minimum distance 0 so semi major axis = R/2. Hence if t is the time of fall then 2 3 2 R/2 22 7 (3) -(@) « ere or t= T/4V2 = 365 / 4V2 = 64:5 days. 1.204 T= 22R°?/Vym, If the distances are scaled down, R°” decreases by a factor 1°and so does m, . Hence T does not change. mm, 1.205 The double star can be replaced by a single star of mass m,+m, moving about the centre of mass subjected to the force y m, m,/1°. Then 2xF? ie 2nras T= So ( i or, r= 1.206 (a) The gravitational potential due to m, at the point of location of mz : Otek ce Yan f Bare f Uae 1 z ; mm So, Uy, = m,Vy= 17% Similarly ee mm T 104 1.207 cae m me ym, o——_.,—___+ -—----—| }—-> Uy= Uy,= U= im rox Ik xe dx (b) Choose the location of the point mass as the origin. Then the potential erfergy dU of an element of mass amt = Mae of the rod in the field of the point mass is Mod dU =~ ym Fae where x is the distance between the element and the point. (Note that the rod and the point mass are on a straight line.) If then a is the distance of the nearer end of the rod from the point mass. mM{ dx M at bla 4) The force of interaction is ou fe ag 2 iM ee 1 | | alas) 1 z)- ynM 14+-\ * @ Minus sign means attraction. As the planet is under central force (gravitational interaction), its angular momentum is conserved about the Sun (which is situated at one of the focii of the ellipse) an So, MVr,= MVzr, OF, ye a q@) 4 From the conservation of mechanical energy of the system (Sun + planet), ymm 1 ymm 1 2 a7 tymye - tam, or, atm 12 (tm) 1.2 rysing (1)] : 1 2 ?P nae Thus, vy V2ym, 7,1 7,(r, +7) (2) Hence Mx myyry= mV2ym, ry F2/ (r; +7) 1.208 1.209 105 From the previous problem, if r, , r, are the maximum and minimum distances from the sun to the planet and v, , v2 are the corresponding velocities, then, say, E= dmv} - = Vie = TO PUsing Eq. (2) of 1.207] ole et ee ee ntn mm Mh hth 2a where 2d = major axis =r, + r,. The same result can also be obtained directly by writing an equation analogous to Eq (1) of problem 1.191. 1 2, Me ymm, E= gmre+ ar (Here M is angular momentum of the planet and m is its mass). For extreme position r= 0 and we get the quadratic Me Er +ymmy - m7? The sum of the two roots of this equation are ths E724 Thus er 2a From the conservtion of angular momentum about the Sun. MvorySiNA= MV, T= MVZr, Of, VyTL= Vyl,= Vor2 Sina () From conservation of mechanical energy, m m,m sey eee 1my-t = Qe 2 Ty - 1m, vorsivoa ym, | i 20% 2n " 2ym, or, (#- t "Ae 2umn-sirina 0 0 7%) 7m, 2% . @-9) ™) Ym, where 1 = ¥7y/ym,, (1m, is the mass of the Sun). v7 sin” 2 i 2) r[12V¥1-@-mnsina | 106 1.210 1211 At the minimum separation with the Sun, the cosmic body’s velocity is perpendicular to its position vector relative to the Sun. If r,,, be the sought minimum distance, from con- servation of angular momentum about the Sun (C). Vol mvp = Mvraig OF Ve @ ‘min From conservation of mechanical energy of tlie system (sun + cosmic body), mim Forge may? 2 ‘min 2 2 Yo Ym, % 5 So, Zz Tan 22, (using 1) or, Vo rg + 2Y My Fain — Vg? = 0 : = 2y m,2V4y? m? + 4v2 v2 0? -ym,xVy? m2 + ve? 0, Tig ye 2ve ve Hence, taking positive root Tain (Ym, /ve)[ VI (10g/ym,¥ - 1] Suppose that the sphere has a radius equal to a. We may imagine that the sphere is made up of concentric thin spherical shells (layers) with radii ranging from 0 to a; and each spherical layer is made up of elementry bands (rings). Let us first calculate potential due to an elementry band of a spherjcal layer at the point of location of the point mass m (say point P) (Fig.). As all the points of the band are located at the distance / from the point P, so, ape - 1a (where mass of the band) @ 7 | (2x asin 8) (add) am ( na | Ana = ( S*) sino ao (® And 1? = a? +7r?—2arcos® (3) Differentiating Eq. (3), we get Idl = ar sin@ dO @ Hence using above equations aon -(Sar ja ©) 1.212 1.213 107 Now integrating this Eq. over the whole spherical layer au donfoge rat f So dg- at © Equation (6) demonstrates that the potential produced by a thin uniform spherical layer outside the layer is such as if the whole mass of the layer were concentrated at it’s centre; Hence the potential due to the sphere at point P; (M_ = fap~ -tfam--¥ 0) This expression is similar to that of Eq. (6) Hence thte sought potential energy of gravitational interaction of the particle m and the sphere, = mon tli U= mp - (b) Using the Eq,, G,= -42 G,=-YF (wing Ea. 7) So = MF and Fe mont ®) r (The problem has already a clear hint in the answer sheet of the problem book). Here we adopt a different method, Let m be the mass of the spherical layer, wich is imagined to be made up of rings. At a point inside the spherical layer at distance r from the centre, the gravitational potential due to a ring element of radius a equals, age _ dl (see Eq. (5) of solution of 1.211) Hence a or Hence gravitational field strength as well as field force becomes zero, inside a thin sphereical layer. One can imagine that the uniform hemisphere is made up of thin hemispherical layers of radii ranging from 0 to R. Let us consider such a layer (Fig.). Potential at point O, due to this layer is, 108 1.214 2 d= ~ Mt. 3M yar, where dm= —M__| 4" | g, [ R (2/3) aR? \ 2 (This is because all points of each hemispherical shell are equidistant from O.) R 3yM IM. Hence, g= fap~ - "AE frir= -2 0 M Hence, the work done by the gravitational field VN force on the particle of mass m, to remove it to infinity is given by the formula m A= mg, since @ = 0 at infinity. \] Hence the sought work, y)) Ap... 2M. a re (The work done by the external agent is — A.) In the solution of problem 1.211, we have obtained and G due to a uniform shpere, at a distance r from it’s centre outside it. We have from Eqs. (7) and (8) of 1.211, p= ut and G= ur (A) Accordance with the Eq. (1) of the solution of 1.212, potential due to a spherical shell of radius a, at any point, inside it becomes p= @.. const. and G,= -22~ 0 ®) a or For a point (say P) which lies inside the uniform solid sphere, the potential ¢ at that point may be represented as a sum. = Oth, where @, is the potential of a solid sphere having radius r and @, is the potential of the layer of radii r and R. In accordance with equation (A) + Mad *)- Wr =-t sur %1° ~F\Ga73) aR? 3 R The potential @, produced by the layer (thick shell) is the same at all points inside it. The potential @, is easiest to calculate, for the point positioned at the layer’s centre. Using Eq. ®) R =- aM __3 YM p2_,2 = if : 2 ek ry) : where dM = —! gn rtdr= (3M) 2 a (4/3) xR R is the mass of a thin layer betveen the radii + and r+ dr. 2 M Thus oe wens (Se)b-Z) © 1.215 109 From the Eq. G- a2 yMr G,= 3 =~ Mo Anos or G= -ESTS - 15 n0F (here p= AnR3 3 , is the density of the sphere) ©) The plots @ (r) and G (r) for a uniform sphere of radius R are shown in figure of answersheet. Alternate : Like Gauss’s theorem of electrostatics, one can derive Gauss’s theorem for gravitation in the form f G+ dS -4 RY Mciocea » For calculation of G at a point inside the sphere at a distance r from its centre, let us consider a Gaussian surface of radius r, Then, G,4xr= ~4ar(tey or, G= “1, So yMie 4 M Hence, Ge -IMpe -y4npr{as p» —“, R? 73 or c aaa) So, s-Jow-f- ti ras ft Integrating and summing i we get, - oie °""2R ( 2] And from Gauss’s theorem for outside it : G4ur7= -4nyM or G,= pals r Thus ot f Gar -u r Treating the cavity as negative mass of density - p in a uniform sphere density +p and using the superposition principle, the sought field strength is : ee or G= ~ dni + -Frx-p) wr (where 77 and 7~ are the position vectors of an orbitrary point P inside the cavity with respect to centre of sphere and cavity respectively.) Thus Gea -Snyp(7-7)= -4nypl 3 TYP (7-7) = -3%YP 110 1.216 1217 We partition the solid sphere into thin spherical layers and consider a layer of thickness dr lying at a distance r from the centre of the ball. Each spherical layer presses on the layers within it. The considered layer is attracted to the part of the sphere lying within it (the outer part does not act on the layer). Hence for the considered layer dp4nr?=dF 4s v(3*" )axrtars) r (where p is the mean density of sphere) or, dP4nr?= or, dp= fay ptrdr R 2 Thus p= f ap- ye? (R?-1?) 7 (The pressure.must vanish at r = R.) oe pe 200 = (?/R%) 7M] a4, Potting p= M/(4/3) xR? Putting r= 0, we have the pressure at sphere’s centre, and treating it as the Earth where mean density is equal to p= 5-5 x 10° kg/m? and R= 64x 107 km we have, p= 1-73x10"Pa or 1:72 x 10° atms. (a) Since the potential at each point of a spherical surface (shell) is constant and is equal to p= - a [as we have in Eq. (1) of solution of problem 1.212] We obtain in accordance with the equation v= Lange bef m a (- 3) me R 2R (The factor 5 (b) In this case the potential inside the sphere depends only on r (see Eq. (C) of the solution of problem 1.214) is needed otherwise contribution of different mass elements is counted twice.) 2 3ym i= 2R 3R? Here dm is the mass of an elementry spherical layer confined between the radii rand r+dr: g-- 2 3m) 2 dm= (4nr?drp)= (a) dr 1.218 1.219 1.220 Mi Letw= V SE = circular frequency of the satellite in the outer orbit, r coo= WE, = circular frequency of the satellite in the inner orbit. (r-Are So, relative angular velocity = w# cw where — sign is to be taken when the satellites are moving in the same sense and + sign if they are moving in opposite sense. Hence, time between closest approaches -—~2% 2 2x 1 +o 3Ar ®o Vy, [127 +8 where 6 is 0 in the first case and 2 in the second case. { 45 days (6 = 0) 0-80 hour (8 = 2) YM _ 667x107! x 5-96 x 10% oo = 9-8 m/s* eR (637 x10 2 2 ~ otRe (22) pe (—2%22 ' cs 2 @,= wR (F)* (sexssnox7 7) 637%10 0-034 m/s’ M “Ay a ge eee COP nAO X197 X10" gg 1073 m/s? Re (149-50 x 10° x 10°) Then @, : @) 3 @; = 1: 0-0034 : 0-0006 Let h be the sought height in the first case. so ory Me 100 8 (R+hy —M_,. 2, (143) (+3) R R 112 1.221 1.222 1.223 -2 or 1+> 2 i R From the statement of the problem, it is obvious that in this case h< = abk+AB(-k)= (ab-AB)k @) Resultant of the external force ee Fa F,+Fy= Aj+Bi 2) As N-F= 0 (as N1F) so the sought arm / of the force F VA? +B? oe For coplanar forces, about any point in the same plane, 1x Fy=7xF (where _ = > F. = resultant force) or, Nv, =r F. Nye Thus length of the arm, [= =" Frye Here obviously [F,, |= 2F and it is directed toward right along AC. Take the origin at C. Then about C, Ne (VFaF+ yer aF | directed normally into the plane of figure. (Here a = side of the square.) Thus N= F Jy directed into the plane of the figure. F(aN2)_ a cS Hence 1m TUE Se = F sin 4s Thus the point of application of force is at the mid point of the side BC. 1238 1.239 119 (a) Consider a strip of length dx at a perpendicular distance x from the axis about which we have to find the moment of inertia of the rod. The elemental mass of the rod equals m dm= 1 & Moment of inertia of this element about the axis d= dmx? = Fedex? Thus, moment of inertia of the rod, as a whole about the given axis T m2 mi? tof Baten ME ° (b) Let us imagine the plane of plate as xy plane taking the origin at the intersection point of the sides of the plate (Fig.). 0 Obviously I= f amy? Ser : m 2 : f ( ab ee )y oO ee ma = 3 a Similarly I= me Hence from perpendicular axis theorem L=I,tl,= 5 (27+7), which is the sought ,moment of inertia. (a) Consider an elementry disc of thickness dx. Moment of inertia of this element about the z-axis, passing through its C.M. dm) R* R? — dl,= (am) R. pSar & — a where p = density of the material of the plate and S = area of cross section of the plate. Thus the sought moment of inertia b b 2 2 tp 25 face Bose 2 2 0 - FPR‘ (asS= =R?) 120 putting all the vallues we get, J, = 2: gm-m? (b) Consider an element disc of radius r and thickness dx at a distance x from the point O. Then r= x tana and volume of the disc = me? tan? ade Hence, its mass dm= xx” tana. dep (where Moment of inertia of this element, about the axis OA, 3 = density of the cone = m/ExR7h A | a ae us SSS t= dm 2 2 = (mx? tana de) 2 No 7 ape ‘tan ‘ade h Thus the sought moment of inertia J= 2 tanta. f ade Hence 1.240 (a) Let us consider a lamina of an arbitrary shape and indicate by 1,2 and 3, three axes coinciding with x, y and 2- axes and the plane of lamina as x - y plane. Now, moment of inertia of a point mass about x= axis, dl, = dmy? 4430) Thus moment of inertia of the lamina about this axis, I, = f dmy? Similarly, J, = fame? and I, = f dmr? = fidm(x?+y?) as r= Vx74y7 Thus, I= 1,+1, 05, b= 14h, (b) Let us take the plane of the disc as x -y plane and origin to the centre of the disc (Fig.) From the symmetry J, = J,. Let us consider a ring element of radius r and thickness dr, then the moment of inertia of the ring element about the y— axis. 1.241 121 dl, = dmr? = a (Dara)? Thus the moment of inertia of the disc about z— axis R 2m 3 mR? 1 Bf ra ae But we have I= 1,+1,= U, I, mR? Thus i= 27 47 For simplicity let us use a mathematical trick. We consider the portion of the given disc as the superposition of two. complete discs (without holes), one of positive density and radius R and other of negative density but of same magnitude and radius R/2. As (area) a (mass), the respective masses of the considered discs are (4m /3) and (- m/3) respectively, and these masses can be imagined to be situated at their respective centers (C.M). Let us take point O as origin and point x - axis towards right. Obviously the C.M. of the shaded position of given shape lies on the x - axis. Hence the C.M. (C) of the shaded portion is given by (-m/3)(-R/2)+(4m/3)0 p (-m/3)+4m/3 "6 Thus C.M. of the shape is at a distance R/6 from point O toward x - axis Using parallel axis theorem and bearing in mind that the moment of inertia of a complete homogeneous disc of radius mp and radius ry x= equals }-mig1r3. The moment of inetia of the small disc of mass (-m/3) and radius R/2 about the axis passing through point C and perpendicular to the plane of the disc z-4(-™)(R) ,(-™)(2,8 wr 2 2 2 = _. oka Ae 24 2 2 Similarly Le a(S )e+(F)(8) 2 aR? 4 mR? mmr + 27 122 1.242 Moment of inertia of the shaded portion, about the axis passing through it’s certre, Now, if R = r + dr, the shaded portion becomes a shell, which is the required shape to calculate the moment of inertia. Now, I= 2-Snp|(r+dr)5-75} : Boa p(r+5rtdr+ -*) Neglecting higher terms. : 3 (41? drp )r? = Zor? 1.243 (a) Net force which is effective on the system (cylinder M + body m) is the weight of the body m in a uniform gravitational field, which is a constant, Thus the initial acceleration of the body m is also constant. From the conservation of mechanical energy of the said system in the uniform field of gravity at time t= Ar :AT+AU = 0 1a LMR? 2 or zm +z 8 -mgAh=0 or, 4 (2m+M)v?— mg dh= 0 [as v= oR at all times ] But v? = 2w Ah Hence using it in Eq. (1), we get Z eeamgee 4 (2m +M ) 2w Ah — mg Bh= 0 of w (im+M) w 2m; From the kinematical relationship, B = = tener Thus the sought angular velocity of the cylinder See o()= B= GoM)R'” (1sM/im)R (b) Sought kinetic energy. 2 T(t)= dm? AME 2 5 (2m+M)R? 0? 1.244 1.245 1.246 123 For equilibrium of the disc and axle 2T = mg or T= mg/2 As the disc unwinds, it has an angular acceleration B given by B= 21r or B= a mer I The corresponding linear acceleration is 2 rB= w= Me G Since the disc remains stationary under the combined action of this acceleration and the acceleration (-w) of the bar which is transmitted to the axle, we must have mg mgr I Let the rod be deviated through an angle q’ from its initial position at an arbitrary instant of time, measured relative to the initial position in the positive direction. From the equation of the increment of the mechanical energy of the system. we AT= Ang i or, zie = fN,do : 2 or, IMI" 22 [Freospap = Flsing -\ / SF sing Thus, o MI First of all, let us sketch free body diagram of each body. Since the cylinder is rotating and massive, the tension will be different in both the sections of threads. From Newton’s law in projection form for the bodies m, and m, and noting that w, = w,= w= BR, (as no thread slipping), we have (m, > m,) : m,g-T,= m,w= m, BR g and T,-m,g= m,w Qa) @) Now from the equation of rotational dynamics , of a solid about stationary axis of rotation. i.e. qh 2 N,= IB, for the cylinder. or, (T,-T))R= 1B = mR? B/2 @ Tb 12 Similtancous solution of the above equations yields : iy wy wf bmnm)8 yg Hi mms am) . dat i a(meme) 7 (a) mg. mag. 124 1.247 1.248 As the systemt (m +m, +m, ) is under constant forces, the acceleration of body m, an ‘m, is constant. In addition to it the velocities and accelerations of bodies m, and m, at equal in magnitude (say v and w) because the length of the thread is constant. From the equation of increament of mechanical energy i.e. AT + AU = A,,, at time ¢ whe block m, is distance h below from initial position corresponding to t= 0, 1 1 (mR?) v? pm) Omg — km, gh a (as angular velocity = v/R for no slipping of thread.) But v? = 2wh So using it in (1), we get 2 (m, - km, ) 8 Ae er ied 2 m+2(m,+m,) Thus the work done by the friction force on m, 1 Ay, = — kanygh = tm (5 0?) fam, (my ~ kom, ) g? 1? : ” ~n+2.(m, +m) (using 2). In the problem, the rigid body is in translation equlibrium but there is an angular retardation. We first sketch the free body diagram of the cylinder. Obviously the friction forces, acting on the cylinder, are kinetic. From the condition of translational equlibrium for the cylinder, mg= N,+kN,; N= kN, mj mj Hence, N= reac Ny= ee For pure rotation ‘of the cylinder about its rotation axis, N, = JB, mR? or, - kN, R-kN,R= > B, KNg; kingR(1 +k) mR? eee a or, B,= - aan aah R + = KN, Now, from the kinematical equation, 7 = w) +28, AQ we have, wg (1+k?)R HCisEyg eae w= 0 125 Hence, the sought number of turns, So. wg (1 +k? yR In Bxk(1+k)g 1.249 It is the moment of friction force which brings the disc to rest. The force of friction is applied to each section of the disc, and since these sections lie at different distances from the axis, the moments of the forces of friction differ from section to section. To find N,, where z is the axis of rotation of the disc let us partition the disc into thin rings (Fig.). The force of friction acting on the considered element dfr= k(2nrdro)g, (where o is the density of the disc) The moment of this force of friction is dN, = -rdfr= -2nkogr’dr Integrating with respect to r from zero to R, we get n= 2 2 : N,=-2nkog fr dr= -2nkogR®. z oO For the rotation of the disc about the stationary dr axis z, from the equation N, = 1B, Ey Thus from the angular kinematical equation @,= Og, + Bt = __ dk _ FR 0 n+ aR |f orf ake 1.250 According to the question, 122. Wa ot, I= Bm ~ kat Vo. a it Integrating, Vo = -57 + Van 2,2 or, o= ua Oa 4, (Noting that at f= 0, w= W) 2Ve@ k Let the flywheel stops at t= fo then from Eq. (1), f= Hence sought average angular velocity 2VOo k - S(S eH say) a oe 2 VO 3 k fa 126 1.251 1.252 aM, Letus use the equation =" = N,relative to the axis through O 0) For this purpose, let us find the angular momentum of the system M, about the given rotation axis and the corresponding torque N,, The angular momentum is M, = Iw + mvR = (Fema [where J = oR? and v= wR (no cord slipping)] So, i pre zee (Mqoema’), @ The downward pull of gravity on the overhanging part is the only external force, which exerts a torque about the z-axis, passing through O and is given by, m sh . aM, Hence from the equation Ns 2 (‘f oma?) FxgR age Thus, 8." Rar eam)? ® Note : We may solve this problem using conservation of mechanical energy of the system (cylinder + thread) in the uniform field of gravity. (a) Let us indicate the forces acting on the sphere and their points of application. Choose positive direction of x and @ (rotation angle) along the incline in downward direction and in the sense of @ (for undirectional rotation) respectively. Now from equations of dynamics of rigid body i.e. F,= mw,, and N.,= I, B, we get : mg sina- f,= mw qa) and frR= 2 mk 26 2) But firs kag cosa. N fr @) In addition, the absence of slipping provides 9 mA r the kinematical realtionship between the accelerations : £) w= BR o® The simultaneous solution of all the four ‘on equations yields : Lg kcosaz 2 sin a, or kz tuna (b) Solving Eqs. (1) and (2) [of part (a)], we get : 1.253 1.254 127 a wo= 7esina. As the sphere starts at t= 0 along positive x axis, for pure rolling v, (t)= wt Sgsinar () Hence the sought kinetic energy ee eee 2 = Tm pvt 7 mR? w= To mv. (as o= v/R) 15 5 Ply 2 He or 75m (esas) 7a me sin’ at (a) Let us indicate the forces and their points of application for the cylinder. Choosing the positive direction for x and @ as shown in the figure, we write the equation of motion of the cylinder axis and the equation of moments in the C.M. frame relative to that axis ie. from equation F, =, mw, and N,= I, B, 2 mg-2T = mw,; 27R= 7p As there is no slipping of thread on the cylinder w= BR From these three equations = mee 22s. 2 T= Gn 13N, B= 2 5x 10? rad/s (b) we have B= 28 So, w, = 2g>0 or, in vector form W, = ay Pe F-v™ F (61) = mz 6 #1)= mgt Let us depict the forces and their points of application corresponding to the cylinder attached with the elevator. Newton’s second law for solid in vector form in the frame of elevator, gives : 2+ mg’t m(- wh) = mis” Q) The equation of moment in the C.M. frame relative to the cylinder axis i.e. from N,= 1,8, - [as thread does not slip on the cylinder, w= BR ] 128 1.255 1.256 mw or, T-"t As (I) TH? so in vector form 7 mw 7. -% Q) Solving Eqs. (1) and (2), # = 2 @ wh) and sought force oot lk Fa 2T= 5m@ Wi). Let us depict the forces and their points of application for the spool. Choosing the positive direction for x and @ as shown in the fig., we apply F,= mw,, and N.:= I, B, and get 0 mg sina -~T= mw; Tr= IB “Notice that if point of a solid in plane motion is connected with a thread, the projection of velocity vector of the solid’s point of contact along the length of the thread equals the velocity of the other end of the thread (if it is not slacked)” Thus in our problem, v,= vg but vo= 0, hence point P is the instantaneous centre of rotation of zero velocity for the spool. Therefore v, = or and subsequently w,= Br. MW Solving the equations simultaneously, we get = SEAR = 16 m/s? : 1+, ig. mr Let us sketch the force diagram for solid cylinder and apply Newton’s second law in projection form along x and y axes (Fig.) : Jr, + frz= mw, @) and N,+N,-mg-F=0 or N,+N,= mg+F 2) Now choosing positive direction of p as shown. in the figure and using N.,= 1, Bp we get 2 2 FR- (fr, + fr)R= me Be Ree @) [as for pure rolling w, = BR J. In addition to, fry + fry sk (N, + No) (4) 1.257 1.258 129 Solving the Eqs., we get re 2kme yp pn 3kme @-3%) mx 23k kW, +Ny) wn ways et k k 3kmg)_ _2ke 7 pq U8 + Fax] ened 2-3k (a) Let us choose the positive direction of the rotation angle @, such that w,, and B, have identical signs (Fig.). Equation of motion, F, = mw,, and N,, = I. B, gives : F cosa - fr= mw,: frR-Fr= I,B,= ymR?B, In the absence of the slipping of the spool w,,= B,R From the three equations w,,= w,= P1SS2=C/R)I | where cos a> = (a) m(1+y¥) vy R (b) As static friction (fr) does not work on the spool, from the equation of the increment of mechanical energy A, = AT. Aue Smid ym? % tmasne : Fmt +y) 20, xe Fm(L+y2m (im 4 ") 2 Piles 2 fr “2m +9) ™g. Note|that at cos a= r/R, there is no rolling and for cos &<1r/R, Wz,<0, i.e. the spool will move towards negative x-axis and rotate in anticlockwise sense. For the cylinder from the equation N, = IB, about its stationary axis of rotation. 2Tr= Mp eS p- 2 Q) For the rotation of the lower cylinder from the equation N., = I. B, mr oy Dake 2Tr= - o, B= 7 =B Now for the translational motion of lower cylinder from the Eq. F,= mw,. mg -2T= mw, 2) As there is no slipping of threads on the cylinders : w= Bir+ Br= 2Br @ 130 1.259 1.260 Simultaneous solution of (1), (2) and (3) yields me T 10° Let us depict the forces acting on the pulley and weight A, and indicate positive direction for x and @ as shown in the figure. For the cylinder from the equation F,= m wy and N.,= T,B, we get Mg+T,-2T= Mw, Q) Iw, and 27R +T,(2R)= IB= = 2 | For the weight A from the equation F,= mw, mg -T,= mw, Q) As there is no slipping of the threads on the pulleys. = w,+2BR=w,+2w.=3w, (4) Simultaneous solutions of above four equations gives : 3(M+3m) We a (some R (a) For the translational motion of the system (m,+ m,), from the equation : F,= mwa, F= (m+myw, or, w.= F/(m, +m) (ty Now for the rotational motion of cylinder from the equation : N.,= 1, B, Pr Fre 5 ot bra = Q) But w= Ww, + Br, So F_.2F_ F(3m,+2m) +e m, +m, my m (m, +m) (b) From the equation of increment of mechanical energy : AT= A,, Here AT= T(t), so, T= Aggy As force F is constant and is directed along x-axis the sought work done. Aas Fx (where x is the displacement of the point of application of the force F during time interval t) 1,261 1.262 131 1, 2). PE @ms2m) . F( Ze * 2m, (m, +m) (using Eq. G) Alternate : T(t) = Trpansiauon (t) + Trotaugs (2) =T() Ea (Ft 1m? Fe 7 Gm +2m) 20m9m)(jarmy) *2 2 ( ) 2m, (m, m,) mr Choosing the positive direction for x and @ as shown in Fig, let us we write the equation of motion for the sphere F, = mw,, and N,.=1,B, fr=m,w,; frr=aZm,? 6 (wy is the acceleration of the C.M. of sphere.) For the plank from the Eq. F, = mw, F-fp=m,™, In addition, the condition for the absence of slipping of the sphere yields the kinematical relation between the accelerations : w=, +Br Simultaneous solution of the four equations yields : and wom my + 5m, (a) Let us depict the forces acting on the cylinder and their point of applications for the cylinder and indicate positive direction of x and p as shown in the figure. From the equations for the plane motion of a solid F,= mw,, and N,, = I,B,: kmga= mw, or w= kg (a 2 —kng R= ma or B= -248 Q) Let the cylinder starts pure rolling att = fy after releasing on the horizontal floor at ¢ = 0. From the angular kinematical equation or o- ay-2 Me @) Wo From the equation of the linear kinematics, \ WMMTT T177 Vex Voce + Wey t or v= O+kG fy (4) 132 1.263 1.264 But at the moment ¢= f, when pure rolling starts v,= wR so, kgty= (o0-2 E40} R Thus f= aE (b) As the cylinder pick, up speed till it starts rolling, the point of contact has a purely translatory movement equal to hw, in the forward directions but there is also a backward movement of the point of contact of magnitude (19%) - 5 B2)R. Because of slipping the net displacement is backwards. The total work done is then, Ag = mg (>. 8 (ogo+t wap] = kmg [#8 . {-7e) gR- onto] RK [OR o, sigs | 6 3 The same result can also be obtained by the work-energy theorem, Ay = AT. Let us write the equation of motion for the centre of the sphere at the moment of breaking-off: mv?/(R +1) = mg cos 8, where v is the velocity of the centre of the sphere at that moment, and 0 is the corresponding angle (Fig.). The velocity v can be found from the energy conservation law : A 1g? mgh= > mv? + 5 Tor, where J is the moment of inertia of the sphere relative to the axis passing through the sphere’s 2 5 v= or; h= (R+r)(1-cos 8). centre. ie. [= =mr’. In addition, From these four equations we obtain w= V10g(R+n)17P. Since the cylinder moves without sliding, the centre of the cylinder rotates about the point O, while passing through the common edge of the planes. In other words, the point O becomes the foot of the instantaneous axis of rotation of the cylinder. It at any instant during this motion the velocity of the C.M. is v, when the angle (shown in the figure) is B, we have my R = mgcosB-N, 1.265 133 where N is the normal reaction of the edge 2 NR - eee 7 or, vw gRoosp-"R ty From the energy conservation law, 2 4 oe plo z to ze mg R (1 - cos B) 2 But y= ema? = 3 mk’, (from the parallel axis theorem) x Thus, Ya +S gR (1 - cos) 2 From (1) and (2) Rm Zereosp-4) - The angle f in this equation is clearly smaller than or equal to o so putting B = a we get = Rercosa—4) - MR where No is the corresponding reaction. Note that N 2 No. No jumping occurs during this turning if Ny > 0. Hence, vy must be less than Yaar VEE (1 008 4 4) Clearly the tendency of bouncing of the hoop will be maximum when the small body A, will be at the highest point of the hoop during its rolling motion. Let the velocity of C.M. of the hoop equal v at this position. The static friction does no work on the hoop, so from conservation of mechanical energy; E, = Ey 7 2 L 1 iR?(%) —-mgR= Emveet 1 pelY or, O+zmetzmR ( mgR zm (2vy tymvsomR (i) +mgR or, 3v= e-2gR aw From the equation F, = mw, for body A at final position 2 : 2 mer mo? R= m(R] R (2) a 134 1.266 1.267 1.268 As the hoop has no acceleration in vertical direction, so for the hoop, N+N'= mg @) From Egg. (2) and (3), N= 2mg- av 4 As the hoop does not bounce, Nz 0 (5) So from Eqs. (1), (4) and (5), 8gR-v% R20 of 8gR= w Hence vos V8gR Since the lower part of the belt is in contact with the rigid floor, velocity of this part becomes zero. The crawler moves with velocity v, hence the velocity of upper part of the belt becomes 2v by the rolling condition and kinetic energy of upper part 7 ; 7 (2v)’ = mv’, which is also the sought kinetic energy, assuming that the length of the belt is much larger than the radius of the wheels. The sphere has two types of motion, one is the rotation about its own axis and the other is motion in a circle of radius R. Hence the sought kinetic energy 1 2 where J, is the moment of inertia about its own axis, and J, is the moment of inertia about the vertical axis, passing through O, 1 T= Shojtsho} () But, 1,= Zn? and I, = 2 mr? + mR? (using parallel axis theorem.) Q) In addition to o,= . and ©, = z @) Using (2) and (3) in (1), we get T’ = 3S mi? (: | For a point mass of mass dm, looked at from C rotating frame, the equation is —- a — dmv = f+dmo?r” +2dm(v" x) where 7” = radius vector in the rotating frame with respect to rotation axis and ¥” = velocity in the same frame. The total centrifugal force is clearly ~ ~ Fyy- Sdn?” = mo R, R. is the radius vector of the CM. of the body with respect to rotation axis, also ? > F,,,= Wmv. x@ where we have used the definitions - — mR,= > dm” and mv, = > dmv 1.269 1.270 135 Consider a small element of length dx at a distance x from the point C, which is rotating in a circle of radius r= x sin @ Now, mass of the element = 7 a& ! So, centrifugal force acting on this element - 7 dco" xsin 0 and moment of this force about C, |av |= (7]rorssino-xeoso 2 MO sin 2x7 de and hence, total moment 12 2 mo. Jt pei N of By sin 20 Pde = 57 moo" l? sin 20, 0 Let us consider the system in a frame rotating with the rod. In this frame, the rod is at rest and experiences not only the gravitational force m Z’and the reaction force R, but also the centrifugal force F,,. In the considered frame, from the condition of equilibrium i.e. Ny, = 0 ia or, Nig = mg z sin ® where N,; is the moment of centrifugal force about O. To calculate N,, , let us consider an element of length de, situated at a distance x from the point O. This element is subjected to a horizontal pseudo force 7 dew" xsin ®. The moment of this pseudo force about the axis of rotation through the point O is aNy= (7) dx @? x sin 0.xcos 0 2 = TP sin cos @ x? dx 1 2 272 So ng= f 22 sin cos 07dr= 22 sind cos 0 Q) 0 It follows from Eqs. (1) and (2) that, cos 0 = (ch) or os xo""( 3745) () 2071 207 136 1271 1.272 1.273 When the cube is given an initial velocity on the table in some direction (as shown) it acquires an angular momentum about an axis on the table perpendicular to the initial velocity and (say) just below the C.G.. This angular momentum will disappear when the cube stops and this can only by due to a torque. Frictional forces cannot do this by themselves because they act in the plain containing the axis. But if the force of normal reaction act eccentrically (as shown), their torque can bring about the vanishing of the angular momentum. We can calculate the distance Ax between the point of application of the normal reaction and the C.G. of the cube as follows. Take the moment about Initial velocity —— Axis 1 to the C.G. of all the forces. This must vanish because Initial ae the cube does not turn or turnble on the table. angular eon tables Then if the force of friction is fr momentum a fr 37 N Ax But N= mg and fr= kg, so Ax = ka/2 In the process of motion of the given system the kinetic energy and the angular momentum relative to rotation axis do not vary. Hence, it follows that Mig ge te a Mig 5-00 gM o7P +7) 450 (o is the final angular velocity of the rod) Mi? fe 2 and 370 jy orm o From these equations we obtain 3 o- o/ (3 : or) me v= ool / Vi+3m/M Due to hitting of the ball, the angular impulse received by the rod about the C.M. is equal top i If w is the angular velocity acquired by the rod, we have mi? pl oa Deny KOM q@ In the frame of C.M., the rod is rotating about an axis passing through its mid point with the angular velocity w. Hence the force exerted by one half on the other = mass of one half x acceleration of C.M. of that part, in the frame of C.M. 2 2 F(074}- mot. 2 on 2 4 8 ml 1274 1.275 137 (a) In the process of motion of the given system the kinetic energy and the angular momentum relative to rotation axis do not vary. Hence it follows that 1 wv? = dmv? a(t Jo 2 2 3 and mtn ml Me 2 2.35. From these equations we obtain va (24=4M),, and w = — 3m+ 4M |” 1 +4 m/3M) rn (3ma 4M) > > > As ¥ ¢ fv, so in vector form v = Lory (0) Obviously the sought force provides the centripetal acceleration to ‘the C.M. of the rod and is F,= mW, at 8Mv? 2° 1(1+4M/3my (a) About the axis of rotation of the rod, the angular momentum of the system is conserved. Thus if the velocity of the flying bullet is v. 2 mls (w+ Jo = Mw o- em as mc M Oy) mts I Now from the conservation of mechanical energy of-the system (rod with bullet) in the uniform field of gravity a(m+ we Jo = (tem) gh (1 ~ cosa ) 2) : [because C.M. of rod raises by the height £( 1 cose) ] Solving (1) and (2), we get M V2 in® - V% in »= (5) 38! sing and © 7 sing (b) Sought Ap = [mconem (og) |-m where ol is the veloccity of the bullet and w+ equals the velocity of CM. of the rod after the impact. Putting the value of v and w we get Apa mv = M at sin & This is caused by the reaction at the hinge on the upper end. 138 1.276 1.277 (©) Let the rod starts swinging with angular velocity o’, in this case. Then, like part (a) ( ‘) , 1. 3mvx mune | —+ me? | 00! oF oo! = Mi? Final momentum is 1 5 (Mo Me 3 Pym mxo' + fy a! dyn ol lm F mye yreT 2 tee 3x So, dp~ py-nmmv( 37-1) Poe 2 This vanishes for xeGl (@) As force F on the body is radial so its angular momentum about the axis becomes zero and the angular momentum of the system about the given axis is conserved. Thus ‘R? 2m 5 +m wR? = | 1+ 57 (b) From the equation of the increment of the mechanical energy of the system : AT=A,, 1MR? 2 1(MR® 222 2\ 2 2) 2 +mR )od= Aa Putting the value of w from part (a) and solving we get 2p2 ma R 2m Aue SE (1 22) (a) Let z be the rotation axis of disc and 9 be its rotation angle in accordance with right-hand screw rule (Fig.). (p and g’ are to be measured in the same sense algebraically.) As M, of the system (disc + man) is conserved and M ,; iiia'y ™ 0, We have at any instant, R? q 0-= mae. ‘hom (SE) (SE) a] Lr oe P| m+ (m2) |O* On integrating Jere Sasta)e en or, p= - ? Qa) mtz This gives the total angle of rotation of the disc. 1.278 1.279 139 (b) From Eq, (1) dg__(_™_)ag_ m_\v(t ae m,_)idv(t) Thus the sought force moment from the Eq. N,= 1B, 7 2 oa w- ME Cg. mR) me lsd Veg” NETO 2 at 2 m,|R dt = at > m,m,R Hence N,=- ae (a) Frome the law of conservation of angular momentum of the system relative to vertical axis z, it follows that: Noytheo,= (h+h)o, Hence @,= (1,4, + Ip.) / (1, +) qQ Not that for «, > 0, the corresponding vector @ coincides with the poitive direction to the z axis, and vice versa. As both discs rotates about the same vertical axis z, thus in vector form. = 1,8,+18,/(h+h) However, the problem makes sense only if @, tt @, or @, ft} @ (0) From the equation of increment of mechanical energy of a system: A,, = AT. Using Eq. (1) ata 2 A ~ 2+ hy um Ore) For the closed system (disc + rod), the angular momentum is conserved about any axis. Thus from the conservation of angular momentum of the system about the rotation axis of rod passing through its C.M. gives : ame my 5= m5 +o Q@) 140 1.280 (v’ is the final velocity of the disc and w angular velocity of the rod) For the closed system linear momentum is also conserved. Hence my = my’ +n mv, Q) (where v, is the velocity of C.M. of the rod) From Eqs (1) and (2) we get y= 2 and v- v= ny, Applying conservation of kinetic energy, as the collision is elastic i 1 y2,l. 2,1 nm? > xm gv’? + snmv,) += To (3) or v?— y= 4nv? and hence v+v'= 4, i Then h 4-7 oF | v aq oo Geni c Vectorially, noting that we have taken ¥” parallel to v” - (3 jr 4+ So,” = 0 for n= 4 and 2 4 tv for n>4 Ov 'on See the diagram in the book (Fig. 1.72) (a) When the shaft BB’ is turned through 90° the platform must start turning with angular velocity Q so that the angular momentum remains constant. Here (T4Ig)Q= Inv. 01, Qe 2 q ZUth) B= > If the shaft is tumed through 180°, angular velocity of the sphere changes sign. Thus from conservation of angular momentum, TQ-Ihog= Ip (Here -J, wy is the complete angular momentum of the sphere i. e. we assume that the angular velocity of the sphere is just - «). Then ®o Q= > and the work done must be, $19? +2 h03-Shaz= 1.281 1.282 141 (b) In the case (a), first part, the angular momentum vector of the sphere is precessing with angular velocity Q. Thus a torque, 2 2 0 Ttly is needed, 1,02 The centrifugal force can be calculated by, te xde= > i mlyo? / Then equilibrium, 0 I 4y 2 (h-T) 7 mB > 1 2 and, Ty+T,= 5mho Thus T, vanishes, when ot 8, on VE =6 nds 1 % Then T,= mg7 = 25N See the diagram in the book (Fig. 1.71). (a) The angular velocity @ about OO ' can be resolved into a component parallel to the rod and a component « sinO perpendicular to the rod through C. The component parallel to the rod does not contribute so the angular momentum ee M= Iw sinO pm! @ sin® Also, M,= Msin0 = jmPosin?o This can be obtained directly also, (b) The modulus of M does not change but the modulus of the change of M is | AM |. o : Loa. | AM | = 2Msin(90-0)= 55 m1? wsin20 (c) Here M, = Mcos@ = Iw sin® cos® Now | S| = 1 sind coso 24 = £ mi? 0? sin?o at as M precesses with angular velocity «. 142 1.283 1.284 Here M= Iw is along the symmetry axis. It has two components, the part Iw cos0 is constant and the part M, = Jw sin® presesses, then aM = I@sin@ w' = mgl sind _ — mal = 0-71ad/s (©) This force is the centripetal force due to precession. It acts inward and has the magnitude \Fl- |Z moti = mo!? [sind = 12mN. BF is the distance of the ish clement from the axis. This is the force that the table will exert on the top. See the diagram in the answer shect Ms ML MZ \ ™9 Sce the diagram in the book (Fig. 1.73). ‘The moment of inertia of the disc about its symmentry axis is mR? If the angular velocity of the disc is w then the angular momentum is im R? c, The precession frequency being 2x1, dM@|_1 we have oe | = = mR? wx 2a dt 2 This must equal m(g+w)I, the effective gravitational torques (g being replaced by g +w in the elevator). Thus, on 2291. 500 nat aRon

Das könnte Ihnen auch gefallen